Ajan teoria

Anonyymi

Jos väitän että aikaa ei ole olemassa, miten todistat väitteeni vääräksi?

154

2082

    Vastaukset

    Anonyymi (Kirjaudu / Rekisteröidy)
    5000
    • Anonyymi

      Aika kuluu kun asiat muuttuu. Tai ajalla kuvataan muutoksen määrää.

      Et kai kiistä muutosta? Sitten et kiistä aikaakaan.

      • Anonyymi

        Aika käy pitkäksi, jos ei tapahdu mitään.


    • Anonyymi

      Kun ideaalinen lankapalloheiluri heilahtaa N kertaa, on aikaa kulunut N*T verran, kun T on yhteen heilahdukseen kuluva "aika". Yhtä hyvin voidaan sanoa, että aikaa on kulunut N verran. Tällöin aika on sidoksissa juuri tuohon heiluriin. Heilurissa energia vaihtaa muotoaan liike-energiasta potentiaalienergiaksi. Heilahdukset seuraavat toisiaan niin, että ne voidaan erikseen havaita ja laskea. Mitään aistein havaittava "aikaa" ei näy missään.

      • Anonyymi

        "Mitään aistein havaittavaa "aikaa" ei näy missään."

        Taidat olla oikeassa. Mitään muutoksesta erillistä aikaa ei taida olla.

        Vai kuuluuko aika jos mikään ei muutu? Ainakaan aikaa ei pysty silloin mittaamaan.


    • Anonyymi

      Säteilykvantin energia E = h/T, missä T on värähdysaika ja h on Planckin vakio. Eli värähdysaika T = h/E. Aika ja energia ovat fysiikan kaavoissa kiinteästi sidoksissa toisiinsa.

    • Anonyymi

      Wiki: Vuonna 1967 sekunti määriteltiin jälleen uudestaan atomikellon avulla. Vahvistetun määritelmän mukaan sekunti oli 9 192 631 770 kertaa sellaisen säteilyn jaksonaika, joka vastaa cesium-133-atomin siirtymää perustilan ylihienorakenteen kahden energiatason välillä. Vuonna 2018 määritelmä muotoiltiin sanallisesti uudelleen, mutta aikaisemman kanssa yhtäpitävällä tavalla.

    • Anonyymi

      Fysiikassa (keski)nopeus v = s : t, jossa s on matka ja t matkaan käytetty aika.
      Osoita, ettei kappaleilla ole nopeutta.

      • Anonyymi

        Kyse on kahdesta tapahtumasta: 1) kappale on paikassa A, 2) kappale on paikassa B. Tapahtumien välillä maapallo voi kierähtää esimerkiksi tasan yhden kierroksen itsensä ympäri eli auringon noususta auringon nousuun. A ja B välinen etäisyys voi olla vaikkapa 1000 "jalkaa". Silloin nopeus on 1000 jalkaa yhdessä kierroksessa. Alkuasukkaiden saarella kilpikonna liikkuu ja pallo pyörii. Aikaa ei näy mailla eikä halmeilla eikä sitä kukaan osaa edes kaivata.


      • Anonyymi

        Wiki: Planckin aika on se ajanjakso, joka valonnopeudella kulkevalta fotonilta kuluu Planckin pituuden mittaiseen matkaan, ja sen suuruus on 5,391247 × 10^ − 44 sekuntia. Tämä "ajan kvantti" on lyhyin ajanhetki, jolla on merkitystä Einsteinin suhteellisuusteoriaan perustuvassa maailmankaikkeuden teoreettisessa mallissa.


      • Anonyymi
        Anonyymi kirjoitti:

        Wiki: Planckin aika on se ajanjakso, joka valonnopeudella kulkevalta fotonilta kuluu Planckin pituuden mittaiseen matkaan, ja sen suuruus on 5,391247 × 10^ − 44 sekuntia. Tämä "ajan kvantti" on lyhyin ajanhetki, jolla on merkitystä Einsteinin suhteellisuusteoriaan perustuvassa maailmankaikkeuden teoreettisessa mallissa.

        -Planckin pituus on pituuden yksikkö, joka on noin 1,6 · 10−35 metriä ja on yksi Planckin yksiköistä. Planckin pituus voidaan laskea käyttämällä kolmea luonnonvakiota: valonnopeutta, Planckin vakiota ja gravitaatiovakiota. Planckin pituus on käytännössä lyhin matka jota voidaan käyttää fysiikan laskuissa (ottaen huomioon yleisen suhteellisuusteorian ja Heisenbergin epätarkkuusperiaatteen).


      • Anonyymi
        Anonyymi kirjoitti:

        -Planckin pituus on pituuden yksikkö, joka on noin 1,6 · 10−35 metriä ja on yksi Planckin yksiköistä. Planckin pituus voidaan laskea käyttämällä kolmea luonnonvakiota: valonnopeutta, Planckin vakiota ja gravitaatiovakiota. Planckin pituus on käytännössä lyhin matka jota voidaan käyttää fysiikan laskuissa (ottaen huomioon yleisen suhteellisuusteorian ja Heisenbergin epätarkkuusperiaatteen).

        Vertailun vuoksi atomiytimen koko on suuruusluokkaa 1 · 10^−14 metriä.


      • Anonyymi
        Anonyymi kirjoitti:

        Vertailun vuoksi atomiytimen koko on suuruusluokkaa 1 · 10^−14 metriä.

        Ydin on 10^21 kertaa Planckin pituus, joten ytimen "alapuolella" on vielä reilusti tilaa teoreetikkojen laskelmille.


      • Anonyymi
        Anonyymi kirjoitti:

        -Planckin pituus on pituuden yksikkö, joka on noin 1,6 · 10−35 metriä ja on yksi Planckin yksiköistä. Planckin pituus voidaan laskea käyttämällä kolmea luonnonvakiota: valonnopeutta, Planckin vakiota ja gravitaatiovakiota. Planckin pituus on käytännössä lyhin matka jota voidaan käyttää fysiikan laskuissa (ottaen huomioon yleisen suhteellisuusteorian ja Heisenbergin epätarkkuusperiaatteen).

        Planckin pituus on teoriassa (kahta teoriaa käyttävässä) lyhin matka, jonka voi mitata, (mutta kukaan ei ole yrittänyt). Jos teoriat eivät pidä paikkaansa edes tuhannella Planckin pituudella, ei silloin ole oikeassa olevaa teoriaa, joka sisältäisi määritelmän Planckin pituudelle, joka tulisi sillä perusteella, että se on hiukkanen, jonka Comptonin aallonpituus on sen Schwarzschildin säde. Tämän hetken Planckin pituus on tällöin samanlainen ihmisten valitsema mitta kuin 1 jalka.

        Laskuissa saa onneksi käyttää infinitesimaalista matkaa.


    • Anonyymi

      Aika on fyysikkojen keksintö. Sehän riippuu havainnoitsijan tilasta eli on "venyvä käsite".

    • Anonyymi

      Aika terminä ja käsitteenä on ainoa tapa, miten ymmärrämme ja hahmotamme muutoksia ja kestoa ja koska se on ainoa tapa ja vaikuttanut jo tuhansia vuosia, ollen jokapäivänen asia lähes jokaiseen asiaan liittyvänä, olemme niin sidottuja siihen, että siitä on tullut konkreettinen osa tietoisuutta -> Aikakäsitys -> Tunnemme ajan kulkevan tai kuluvan. Miten aika voisikaan kulkea tai kulua ilman sen keksimistä ja sen toiminnan määrittelyä.

      Jos aikaa ei olisi koskaan keksitty ja kehitetty, emme kokisi nykyään täysin normaaliksi koettuja 'ajallisia etäisyyksiä'. Emme kokisi ajallista etäisyyttä, kun aiheena on esim. toinen maailmansota, tai Rooman valtakunta, tai liitukausi jne. Katso vaikka kuvaa dinosauruksesta ja kuvittele, ettei ole keinoa selittää itselleen, milloin se tallusteli maan päällä. Ajallisen etäisyyden kokemusta ei synny. Koin itse tämän kummallisuuden, kun katsoin yli 100 vuotta sitten kuvattua remasteroitua videota, jossa korjailut ja värit tuovat sen lähemmäs nykyaikaa: https://www.youtube.com/watch?v=0KsJi0UPoH4

      Ajan olemattomuus on helppo perustella:

      1. aikaa ei ole kukaan löytänyt tai havainnut universumissa
      2. millään laitteella ei voi havaita aikaa

      Ja kun kertaa vielä "ajan" historiaa, niin sen olemattomuus vahvistuu. Sitten voikin kysyä, mistä aika tuli. Se tuli samalla logiikalla kuin esim. sentit, tuumat ja kilot. Ajan keksimisen jälkeen tuli myös nopeus.

      Jos joku väittää, että 'aika' on olemassa, voi aina esittää konkreettiset todisteet, missä se on, miten se on, miten se toimii, miten se on havaittu, millä laitteella se on havaittu jne. Mikään teoreettinen ja hypoteettinen selitys ei ole todiste. Samalla voi myös sitten esittää todisteet, mistä ja miten luonnosta löydettiin sentit, tuumat ja kilot.

    • Anonyymi

      Ota suihku esille ja laita vesi suihkuamaan. Ajan suunnan havaitsee siitä, että sen edetessä entropia kasvaa eli vesi hajaantuu suihkusta pisaroiksi jotka putoilevat lattialle. Toiseen suuntaan ilmiö ei tapahdu eli vesi ei itsekseen kylppärin lattialta nouse ja osaa lentää suihkun suuttimeen.

      Aikaa voi mitata entropian lisääntymisen avulla joten aika on olemassa.

      • Anonyymi

        Sanotaan, että aika on rahaa. Mutta raha ei noudata tuota entropiaperiaatetta. Jos näin olisi, niin sosialistin unelma olisi jo kauan sitten toteutunut - kaikki olisivat yhtä köyhiä.


      • Anonyymi
        Anonyymi kirjoitti:

        Sanotaan, että aika on rahaa. Mutta raha ei noudata tuota entropiaperiaatetta. Jos näin olisi, niin sosialistin unelma olisi jo kauan sitten toteutunut - kaikki olisivat yhtä köyhiä.

        Varainjako, jossa kaikilla on yhtäpaljon rahaa on tietyltä entropialtaan tosi pieni. Voisit tietää paljon rahaa satunnaisella valitulla ihmisellä on. Suuressa entropiassa valitulla ihmisellä voi olla mikä raha tahansa yhtä suurella todennäköisyydellä eli et tiedä yhdestä ihmisestä mitään.


      • Anonyymi

        Entropian maksimoitumisperiaate tarkoittaa, että systeemi pyrkii todennäköisimpään tilaansa. Lastenhuoneen tapaksessa se tarkoittaa, että tavarat ovat hujan hajan. Epäjärjestys ei voi enää siitä lisäntyä. Turha siivota, jos tämä ei haittaa.


      • Anonyymi
        Anonyymi kirjoitti:

        Entropian maksimoitumisperiaate tarkoittaa, että systeemi pyrkii todennäköisimpään tilaansa. Lastenhuoneen tapaksessa se tarkoittaa, että tavarat ovat hujan hajan. Epäjärjestys ei voi enää siitä lisäntyä. Turha siivota, jos tämä ei haittaa.

        Jos tavarat ovat hujan hajan, siitä voi päätellä, että siivouksesta on jo jonkin aikaa.


      • Anonyymi
        Anonyymi kirjoitti:

        Jos tavarat ovat hujan hajan, siitä voi päätellä, että siivouksesta on jo jonkin aikaa.

        Kun kahvi kupissa on jäähtynyt huoneen lämpötilaan, niin sen jälkeen ajan kulumista ei voi enää päätellä lämpötilasta.


      • Anonyymi

        Väärin, et mittaa sillä aikaa, vaan mittaat tapahtumaa ajalla.


      • Anonyymi
        Anonyymi kirjoitti:

        Kun kahvi kupissa on jäähtynyt huoneen lämpötilaan, niin sen jälkeen ajan kulumista ei voi enää päätellä lämpötilasta.

        Kun kahvikuppi on jäähtynyt absoluuttiseen nollapisteeseen niin sitten ehkä noin. Maailmankaikkeuden lämpökuolemaan on vielä aikaa. Sitä odotellessa entropian kasvu mittaa aikaa.


      • Anonyymi
        Anonyymi kirjoitti:

        Kun kahvikuppi on jäähtynyt absoluuttiseen nollapisteeseen niin sitten ehkä noin. Maailmankaikkeuden lämpökuolemaan on vielä aikaa. Sitä odotellessa entropian kasvu mittaa aikaa.

        Kyllä huone ja kuppi pysyy noin 293 kelvinissä pitkääään, jos vain polttoaineita piisaa.


      • Anonyymi
        Anonyymi kirjoitti:

        Kun kahvikuppi on jäähtynyt absoluuttiseen nollapisteeseen niin sitten ehkä noin. Maailmankaikkeuden lämpökuolemaan on vielä aikaa. Sitä odotellessa entropian kasvu mittaa aikaa.

        Mikä teitä vaivaa?

        Ajan käsite on hukkunut johonkin,
        Aika on määritelty SI-sekuntina, se on siis tuon mittainen mittayksikkö.

        Se mikä ihmisiä kiinnostaa ajassa, on sen suhde tapahtumiin, mittaamme
        tapahtumia SI-sekunneilla, emme SI-sekuntteja tapahtumilla.
        Paitsi sitä tapahtumaa jolla SI-sekunti määritellään.
        Tietysti voidaan sanoa että mittaamme tapahtumia mittatapahtuman avulla,
        siis sen atomikellon tapahtuman suhteen.

        Se on vain tavattoman työläs tapa, siksi SI-yksiköt on kehitetty että ajan hallinta
        mittauksessa helpottuu.

        Tiede saa sitten tutkia onko "aikaa" jollain tavalla atomitasolla, ainakin se on
        ilmiötasolla ja siitä hieman syvemmällä.

        F.


      • Anonyymi
        Anonyymi kirjoitti:

        Entropian maksimoitumisperiaate tarkoittaa, että systeemi pyrkii todennäköisimpään tilaansa. Lastenhuoneen tapaksessa se tarkoittaa, että tavarat ovat hujan hajan. Epäjärjestys ei voi enää siitä lisäntyä. Turha siivota, jos tämä ei haittaa.

        Systeemin todennäköisin makrotila on vain se, millä on eniten mikrotiloja eli permutaatioita eri mahdollisuuksista. Se ei ole todennäköisin missään dynaamisessa pyrkimisen mielessä, tai kaikki dynamiikka on jopa ensin tukahdutettava. Jos esim. lastenhuoneeseen osuu kohta siivojajärjestön aalto, ei todennäköisyyksiä huomisen huoneen tilasta lasketa mikrotilojen avulla enää. Sitten on vielä erikseen fyysiset systeemit, joissa jotain fysiikkaa täytyy mainita, jotta systeemi pyrkisi mihinkään.

        Varainjaossa 'kaikilla on yhtäpaljon rahaa' koostuu yhdestä mikrotilasta. Kuutiomaisessa lastenhuoneessa 'lamppu on katossa' sisältää 1/5 eri mikrotiloja verrattuna 'lamppu on lattialla tai seinällä' - "epäjärjestykseen" , millä ei tosin ole tekemistä epäjärjestyksen kanssa muutoin kuin silloin, kun kyseessä on todennäköisyys ja et tiedä mikä seinä tai lattia on kyseessä ja voit käyttää vain jotain "epäjärjestys" -sanaa, joka toistaiseksi vaikuttaa hyvin keksityltä. Jotta olisit oikeassa täytyisi sinun myös perustella oikein, että miksi et muka tiedä lampusta joko kaikkea tai vähintään sen seinää kaikissa tapauksissa.


      • Anonyymi
        Anonyymi kirjoitti:

        Kyllä huone ja kuppi pysyy noin 293 kelvinissä pitkääään, jos vain polttoaineita piisaa.

        Kun polttaa polttoaineita tai fuusioi tähdessä vetyä ja heliumia niin entropia lisääntyy...


    • Anonyymi

      Joillakin ihmisillä on kummallinen käsitys, että olisi jotenkin hyvä asia, jos väitettä ei voi todistaa vääräksi.

      Jos väitettä ei millään tavalla voi edes teoriassa todistaa vääräksi, eli se on epäfalsifioitava, se on täysin merkityksetön. Siksi eräs tieteellisen hypoteesin vaatimus on, että on oltava jokin koe- tai havaintoasetelma, jonka jokin tulos/havainto todistaa hypoteesin vääräksi.

      • Anonyymi

        Erinomainen kommentti. Avaus on pelkkää sanahelinää.


    • Anonyymi

      Sitten kun ei enää ole tapahtumia ei ole aikaakaan. Todistettavasti vielä on tapahtumia.

    • Anonyymi

      Ei tätä skeidaa enää

    • Anonyymi

      Aikaa ei ole ilman tapahtumia, eli ajan loppumista "saa odottaa" niin pitkään, että maailmankaikkeuden viimeinen alkeishiukkanen tuhoutuu.

      Aika on tapahtumaketjujen jatkuvaa päivittymistä, eli kaikella on aikansa kuten vanhakansa tietää.

      • Anonyymi

        "Aikaa ei ole ilman tapahtumia"

        Samaa mieltä, mutta toisaalta se ei voi olla tapahtumaketjujen päivitystä,
        oikeastaan todellisuudessa on vain tapahtumia, olivat ne nyt sitten atomikellon
        tai entropiatapahtumia.

        Aika on jotenkin näissä tapahtumissa, silloinkin kun aloitetaan uusi tapahtumaketju,
        aika on heti "mukana" asiassa. Ehkä aika ei ole tapahtuman kantaja, vaan jotenkin
        muuten osallisena.

        F.


      • Anonyymi
        Anonyymi kirjoitti:

        "Aikaa ei ole ilman tapahtumia"

        Samaa mieltä, mutta toisaalta se ei voi olla tapahtumaketjujen päivitystä,
        oikeastaan todellisuudessa on vain tapahtumia, olivat ne nyt sitten atomikellon
        tai entropiatapahtumia.

        Aika on jotenkin näissä tapahtumissa, silloinkin kun aloitetaan uusi tapahtumaketju,
        aika on heti "mukana" asiassa. Ehkä aika ei ole tapahtuman kantaja, vaan jotenkin
        muuten osallisena.

        F.

        Aika on mittayksikkö, ei tapahtuma. Jos ei ole tapahtumia, ei tarvita ajan mittausta. Ajan mittauslaitteiden tapahtumat eivät sinänsä ole aikaa.


      • Anonyymi
        Anonyymi kirjoitti:

        Aika on mittayksikkö, ei tapahtuma. Jos ei ole tapahtumia, ei tarvita ajan mittausta. Ajan mittauslaitteiden tapahtumat eivät sinänsä ole aikaa.

        Aika sellaisenaan ei ole mittayksikkö, SI-sekunti on.
        Aika liittyy jotenkin tapahtumiin, jos ei ole tapahtumia, ei edes voi
        yrittää mitata aikaa, koska kellot tuottavat tapahtumia, eli kellot eivät toimi.

        Niinpä, kellojen tapahtumat eivät ole aikaa, niillä mittaamme muita tapahtumia.
        Ehkäpä aika on kellotapahtumien ja mitattavien tapahtumien suhteessa.

        Kun yleensä mitataan jotakin, mittalaite reagoi mitattavan muutoksiin,
        ajan tapauksessa emme suoraan mittaa aikaa, vaan välillisesti mitattavan
        ja kellon suhteesta päättelemme ajasta jotakin.

        F.


      • Anonyymi
        Anonyymi kirjoitti:

        "Aikaa ei ole ilman tapahtumia"

        Samaa mieltä, mutta toisaalta se ei voi olla tapahtumaketjujen päivitystä,
        oikeastaan todellisuudessa on vain tapahtumia, olivat ne nyt sitten atomikellon
        tai entropiatapahtumia.

        Aika on jotenkin näissä tapahtumissa, silloinkin kun aloitetaan uusi tapahtumaketju,
        aika on heti "mukana" asiassa. Ehkä aika ei ole tapahtuman kantaja, vaan jotenkin
        muuten osallisena.

        F.

        Tapahtumat on sidottu aikaan > aikaa ei ole ilman tapahtumia >>tapahtumia ei ole ilman aikaa.

        Toisaalta jos maailmankaikkeus syntyisi tyhjästä (uudelleen) , aika saataisiin kaupanpäällisenä.

        Näin muotoiltuna ajan ominaisuus olisi ehkä paremmin ymmärrettävissä, uskoisin.


      • Anonyymi
        Anonyymi kirjoitti:

        Tapahtumat on sidottu aikaan > aikaa ei ole ilman tapahtumia >>tapahtumia ei ole ilman aikaa.

        Toisaalta jos maailmankaikkeus syntyisi tyhjästä (uudelleen) , aika saataisiin kaupanpäällisenä.

        Näin muotoiltuna ajan ominaisuus olisi ehkä paremmin ymmärrettävissä, uskoisin.

        "Tapahtumat on sidottu aikaan > aikaa ei ole ilman tapahtumia >>tapahtumia ei ole ilman aikaa."

        Tuo alkaa hipomaan ajan kehämääritelmää, tapahtumat määritellään ajalla
        ja aika määritellään tapahtumilla. Ja vielä negatiivisina määreinä.

        Yleensä aikakeskustelut menevät epämääräisiksi, kun sotkemme toisiinsa
        aikakäsitteen ja ajan mahdollisesti fyysisenä tekijänä, josta emme tiedä oikein
        mitään.

        Tuo yläpuolella oleva tapahtumat ja aika kehitelmä on ajan käsitteeseen
        kuuluva asia, kun puhut ajan ominaisuudesta puhut ajasta itsestään,
        fyysisenä jonakin .

        Tässä voitaisiin erotella tuntematon mahdollinen fyysinen aika, merkitsemällä
        se, aika? kysymysmerkki käy hyvin osoittamaan tuntematonta itse aikaa.
        Aikakäsitteestä tiedämme aika paljon, ajasta? emme oikein mitään.

        Kelloilla on ominaisuutensa, tapahtumilla on ominaisuutensa, mitkä olisi
        ajan? ominaisuudet.
        Jos tietäisimme ajan? ominaisuudet voisimme mitata sitä suoraan aikamittarilla,
        joka ei ehkä olisi kello tavallisessa merkityksessä.

        F.


      • Anonyymi
        Anonyymi kirjoitti:

        "Tapahtumat on sidottu aikaan > aikaa ei ole ilman tapahtumia >>tapahtumia ei ole ilman aikaa."

        Tuo alkaa hipomaan ajan kehämääritelmää, tapahtumat määritellään ajalla
        ja aika määritellään tapahtumilla. Ja vielä negatiivisina määreinä.

        Yleensä aikakeskustelut menevät epämääräisiksi, kun sotkemme toisiinsa
        aikakäsitteen ja ajan mahdollisesti fyysisenä tekijänä, josta emme tiedä oikein
        mitään.

        Tuo yläpuolella oleva tapahtumat ja aika kehitelmä on ajan käsitteeseen
        kuuluva asia, kun puhut ajan ominaisuudesta puhut ajasta itsestään,
        fyysisenä jonakin .

        Tässä voitaisiin erotella tuntematon mahdollinen fyysinen aika, merkitsemällä
        se, aika? kysymysmerkki käy hyvin osoittamaan tuntematonta itse aikaa.
        Aikakäsitteestä tiedämme aika paljon, ajasta? emme oikein mitään.

        Kelloilla on ominaisuutensa, tapahtumilla on ominaisuutensa, mitkä olisi
        ajan? ominaisuudet.
        Jos tietäisimme ajan? ominaisuudet voisimme mitata sitä suoraan aikamittarilla,
        joka ei ehkä olisi kello tavallisessa merkityksessä.

        F.

        Laitanpa vielä yhden selvennyksen, että ei kehäpäättelijäksi minua pääteltäisi.

        Tapahtumat on sidottu aika(viiveeseen) >>>aikaviivettä ei ole jos ei ole ensimmäistäkään tapahtumaa maailmankaikkeudessa >>>>> tapahtumia ei yksinkertaisesti voisi olla jos tätä kyseistä viivettä , aikaa, ei olisi olemassa.

        Joskus olen tosin miettinyt että voisikohan sähkömagneettinen säteily, kuten fotonien liike, tapahtua olla jollain tavalla ajasta erillään. Valonnopeaa liikettä kun on vaikea tulkita "viiveessä liikkumisena"


      • Anonyymi
        Anonyymi kirjoitti:

        Laitanpa vielä yhden selvennyksen, että ei kehäpäättelijäksi minua pääteltäisi.

        Tapahtumat on sidottu aika(viiveeseen) >>>aikaviivettä ei ole jos ei ole ensimmäistäkään tapahtumaa maailmankaikkeudessa >>>>> tapahtumia ei yksinkertaisesti voisi olla jos tätä kyseistä viivettä , aikaa, ei olisi olemassa.

        Joskus olen tosin miettinyt että voisikohan sähkömagneettinen säteily, kuten fotonien liike, tapahtua olla jollain tavalla ajasta erillään. Valonnopeaa liikettä kun on vaikea tulkita "viiveessä liikkumisena"

        Mielenkiintoista.

        Toisaalta, kun määrittelet kahta asiaa toistensa avulla, päädyt aina muna-kana
        ongelmaan. Eli kehämääritelmään.
        Koska kyse on käsitteiden määrittelemisestä, niitä ei pitäisi määritellä
        negatiivisten termien avulla.

        En kiellä ajan olemassaoloa, mietiskelin vain miten aika ja tapahtumat liittyvät
        toisiinsa, toimivaa esimerkkiä ei näytä löytyvän tältäkään palstalta.

        Voisi spekuloida näin.

        -Aika on tapahtumien kantaja.
        -Aika on tapahtumien liimaaja.
        -Aika on tapahtumien synnyttäjä, tämä voi olla ongelmallinen.
        -Tapahtuma on ajan siirtäjä.
        -Tapahtuma on ajan ylläpitäjä.
        -Aika ja tapahtumat ovat elinikäisessä liitossa.
        -Jos tapahtumat loppuvat, aika ei välttämättä lopu, myöhemmin voi tapahtua jotain.


      • Anonyymi
        Anonyymi kirjoitti:

        Mielenkiintoista.

        Toisaalta, kun määrittelet kahta asiaa toistensa avulla, päädyt aina muna-kana
        ongelmaan. Eli kehämääritelmään.
        Koska kyse on käsitteiden määrittelemisestä, niitä ei pitäisi määritellä
        negatiivisten termien avulla.

        En kiellä ajan olemassaoloa, mietiskelin vain miten aika ja tapahtumat liittyvät
        toisiinsa, toimivaa esimerkkiä ei näytä löytyvän tältäkään palstalta.

        Voisi spekuloida näin.

        -Aika on tapahtumien kantaja.
        -Aika on tapahtumien liimaaja.
        -Aika on tapahtumien synnyttäjä, tämä voi olla ongelmallinen.
        -Tapahtuma on ajan siirtäjä.
        -Tapahtuma on ajan ylläpitäjä.
        -Aika ja tapahtumat ovat elinikäisessä liitossa.
        -Jos tapahtumat loppuvat, aika ei välttämättä lopu, myöhemmin voi tapahtua jotain.

        Kerropa vielä lyhyesti:

        -minkälainen on se tapahtuma joka tapahtuu ilman aikaa, eli ajan kulumista kansanomaisesti sanottuna?

        - minkälaista on aika tyhjässä maailmankaikkeudessa jossa ei ole ensimmäistäkään tapahtumaa?

        Siitä vain omin sanoin selostamaan, mielikuvitustahan sulla näyttää riittävän.

        Sen jälkeen voidaan pohtia asiaa eteenpäin ...jos tarvetta siihen löytyy.


      • Anonyymi
        Anonyymi kirjoitti:

        Mielenkiintoista.

        Toisaalta, kun määrittelet kahta asiaa toistensa avulla, päädyt aina muna-kana
        ongelmaan. Eli kehämääritelmään.
        Koska kyse on käsitteiden määrittelemisestä, niitä ei pitäisi määritellä
        negatiivisten termien avulla.

        En kiellä ajan olemassaoloa, mietiskelin vain miten aika ja tapahtumat liittyvät
        toisiinsa, toimivaa esimerkkiä ei näytä löytyvän tältäkään palstalta.

        Voisi spekuloida näin.

        -Aika on tapahtumien kantaja.
        -Aika on tapahtumien liimaaja.
        -Aika on tapahtumien synnyttäjä, tämä voi olla ongelmallinen.
        -Tapahtuma on ajan siirtäjä.
        -Tapahtuma on ajan ylläpitäjä.
        -Aika ja tapahtumat ovat elinikäisessä liitossa.
        -Jos tapahtumat loppuvat, aika ei välttämättä lopu, myöhemmin voi tapahtua jotain.

        "Jos tapahtumat loppuvat, aika ei välttämättä lopu, myöhemmin voi tapahtua jotain"

        Jos noin haluaa ajatella, silloin mikään ei pääty koskaan. Itse kuitenkin ajattelen niin, että jos universumissa kaikki tapahtumat päättyvät absoluuttisesti, myös aika päättyy. Jos sitten tapahtumat käynnistyvät selittämättömästi uudestaan, kysymys on ajankin suhteen sama eli aika ikään kuin syntyy uudestaan.


      • Anonyymi
        Anonyymi kirjoitti:

        Kerropa vielä lyhyesti:

        -minkälainen on se tapahtuma joka tapahtuu ilman aikaa, eli ajan kulumista kansanomaisesti sanottuna?

        - minkälaista on aika tyhjässä maailmankaikkeudessa jossa ei ole ensimmäistäkään tapahtumaa?

        Siitä vain omin sanoin selostamaan, mielikuvitustahan sulla näyttää riittävän.

        Sen jälkeen voidaan pohtia asiaa eteenpäin ...jos tarvetta siihen löytyy.

        "Kerropa vielä lyhyesti:

        -minkälainen on se tapahtuma joka tapahtuu ilman aikaa, eli ajan kulumista kansanomaisesti sanottuna?

        - minkälaista on aika tyhjässä maailmankaikkeudessa jossa ei ole ensimmäistäkään tapahtumaa?"

        Hyviä kysymyksiä, asiaa pitää ehkä taustoittaa hieman että vastaus voisi
        löytyä.

        Jonkinlainen sekaannus vallitsee käsityksessämme tapahtumista.
        Tilannetta voisi verrata rankkitynnyriin josta aiomme saada hyvää juotavaa.

        Rankissa on jo vähän alkoholia, mutta runsaasti muita aineita ehkä myrkkyjäkin.
        Rankkia on suodatettava ja tislattava, puhtaan alkoholin saamiseksi eroon muista
        aineista.

        Ajan suhteen on aika samanlainen tilanne, käsityksemme tapahtumista
        on huonosti jäsentynyttä, vastaten rankkia.
        Aineistoa on suodatettava ja tislattava paremman aikakäsityksen tuottamiseksi.

        Tämä prosessi on alkanut jo tuhansia vuosia sitten egyptiläisten aurinkokelloista,
        jatkuen erityyppisillä kelloilla. Tietenkin myös logiikka, matematiikka yms on
        tuonut lisänsä asiaan.

        Se "tislaus" mitä tapahtumakäsityksestä on saatu aikaan, on otettu käyttöön
        luonnontieteissä, esim geologiassa, käsitys ajasta on saatu paremmaksi ja
        käytännölliseksi.
        Fyysikot ovat menneet omaa linjaansa syvemmälle, he tutkivat kuvaannollisesti
        katsoen alkoholin rakennetta, molekyyleja, atomeja ja alkoholin kvanttiluonnetta
        unohtaen kokonaan alkoholin makrokäyttäytymisen.

        Meillä on paljon tietoa tapahtumista, huonossa tilassa, ajan tislaus on vielä
        kesken, tiedämme jotakin tislauksen tuloksista, mutta emme ajasta? joka olisi
        kaiken perustalla oleva "itse" aika.

        Meillä ihmisillä on siis tapahtumista saatua puhtaampaa aikakäsitystä josta
        fyysikot ovat edenneet etsimään itse aikaa, aikaa?

        Aika? on se perusaika josta emme tiedä oikeastaan mitään, arvailuja on, mutta
        ei oikein käyttökelpoista. Emme voi määritellä tapahtumia ajan? avulla, eikä
        aikaa? tapahtumien avulla.

        Tapahtumista tislattua aikakäsitystä voidaan käyttää luonnontieteissä,
        mutta se anna sellaisenaan tietoa ajasta?

        Miten sitten kysymyksesi, mitä niihin voisi vastata?

        Harmillista kylläkin, kysymyksissä esiintyvä "tapahtuma" on tislautumatonta
        rankkiainesta, josta ei pysty päättelemään mitään ajan suhteen.

        Sellaiseen kysymykseen voisi kylläkin vastata, miten aika oli ennen ihmisen
        ilmestymistä maailmaan?

        Vastaus olisi tislattu ajan käsitys, mutta miten hyvin se olisi ymmärrettävissä,
        ilman että tislaustapahtuman ymmärtää hyvin.

        Täällä näyttää olevan ihmisiä jotka pitävät aikaa vain keinotekoisena
        keksintönä?!

        F.


      • Anonyymi
        Anonyymi kirjoitti:

        "Kerropa vielä lyhyesti:

        -minkälainen on se tapahtuma joka tapahtuu ilman aikaa, eli ajan kulumista kansanomaisesti sanottuna?

        - minkälaista on aika tyhjässä maailmankaikkeudessa jossa ei ole ensimmäistäkään tapahtumaa?"

        Hyviä kysymyksiä, asiaa pitää ehkä taustoittaa hieman että vastaus voisi
        löytyä.

        Jonkinlainen sekaannus vallitsee käsityksessämme tapahtumista.
        Tilannetta voisi verrata rankkitynnyriin josta aiomme saada hyvää juotavaa.

        Rankissa on jo vähän alkoholia, mutta runsaasti muita aineita ehkä myrkkyjäkin.
        Rankkia on suodatettava ja tislattava, puhtaan alkoholin saamiseksi eroon muista
        aineista.

        Ajan suhteen on aika samanlainen tilanne, käsityksemme tapahtumista
        on huonosti jäsentynyttä, vastaten rankkia.
        Aineistoa on suodatettava ja tislattava paremman aikakäsityksen tuottamiseksi.

        Tämä prosessi on alkanut jo tuhansia vuosia sitten egyptiläisten aurinkokelloista,
        jatkuen erityyppisillä kelloilla. Tietenkin myös logiikka, matematiikka yms on
        tuonut lisänsä asiaan.

        Se "tislaus" mitä tapahtumakäsityksestä on saatu aikaan, on otettu käyttöön
        luonnontieteissä, esim geologiassa, käsitys ajasta on saatu paremmaksi ja
        käytännölliseksi.
        Fyysikot ovat menneet omaa linjaansa syvemmälle, he tutkivat kuvaannollisesti
        katsoen alkoholin rakennetta, molekyyleja, atomeja ja alkoholin kvanttiluonnetta
        unohtaen kokonaan alkoholin makrokäyttäytymisen.

        Meillä on paljon tietoa tapahtumista, huonossa tilassa, ajan tislaus on vielä
        kesken, tiedämme jotakin tislauksen tuloksista, mutta emme ajasta? joka olisi
        kaiken perustalla oleva "itse" aika.

        Meillä ihmisillä on siis tapahtumista saatua puhtaampaa aikakäsitystä josta
        fyysikot ovat edenneet etsimään itse aikaa, aikaa?

        Aika? on se perusaika josta emme tiedä oikeastaan mitään, arvailuja on, mutta
        ei oikein käyttökelpoista. Emme voi määritellä tapahtumia ajan? avulla, eikä
        aikaa? tapahtumien avulla.

        Tapahtumista tislattua aikakäsitystä voidaan käyttää luonnontieteissä,
        mutta se anna sellaisenaan tietoa ajasta?

        Miten sitten kysymyksesi, mitä niihin voisi vastata?

        Harmillista kylläkin, kysymyksissä esiintyvä "tapahtuma" on tislautumatonta
        rankkiainesta, josta ei pysty päättelemään mitään ajan suhteen.

        Sellaiseen kysymykseen voisi kylläkin vastata, miten aika oli ennen ihmisen
        ilmestymistä maailmaan?

        Vastaus olisi tislattu ajan käsitys, mutta miten hyvin se olisi ymmärrettävissä,
        ilman että tislaustapahtuman ymmärtää hyvin.

        Täällä näyttää olevan ihmisiä jotka pitävät aikaa vain keinotekoisena
        keksintönä?!

        F.

        E`hei se aika sieltä rankkitynnyristä löydy, ei edes maistelemalla.

        Keskity asian ytimeen eli värähteleekö ne atomit kiljuasatiassa, ajassako värähtelevät vaiko ikuisuudessa,sielläkö se aika luuraa jos on luuraakseen?



        Mietipä sitä aikasi kuluksi.


      • Anonyymi
        Anonyymi kirjoitti:

        E`hei se aika sieltä rankkitynnyristä löydy, ei edes maistelemalla.

        Keskity asian ytimeen eli värähteleekö ne atomit kiljuasatiassa, ajassako värähtelevät vaiko ikuisuudessa,sielläkö se aika luuraa jos on luuraakseen?



        Mietipä sitä aikasi kuluksi.

        Siellä on se "verrata", aivan ymmärrettävää suomea, etkö lukenut tarpeeksi
        tarkasti?


      • Anonyymi
        Anonyymi kirjoitti:

        Aika on mittayksikkö, ei tapahtuma. Jos ei ole tapahtumia, ei tarvita ajan mittausta. Ajan mittauslaitteiden tapahtumat eivät sinänsä ole aikaa.

        Tämä on hyvä kommentti. Ajan pystyy useimmiten poistamaan yhtälöistä matemaattisin keinoin. Joidenkin mielestä tämä tarkoittaa, ettei aikaa ole olemassa. Ennemminkin tämä tarkoittaa, ettei ole olemassa mitään koneistoa, jolla aikaa voisi mitata universaalisti täsmällisesti: Aina on otettava tapahtumien mittakaava huomioon. Tarkkuutta voidaan parantaa tiettyyn rajaan asti. Ajan pienin yksikkö sen sijaan - ongelmana on, että se on monta(kymmentä) dekadia mitttaustarkkuuden alapuolella.


    • Anonyymi

      Ei meidän tarvitse erikseen todistella ajan olemassaoloa. Ei tarvitse tehdä mitään ja silti aloittajan mukaan olemassa olematon aika kuluu ja aloittajan kirjoitus koko ajan peittyy uudemmilla tapahtumilla, joiden avulla aikaa voidaan mitata.

      • Anonyymi

        Siis tarkoitatko, että aika todistaa itsensä olemassaolon? No ei nyt ihan kuitenkaan. Tapahtumilla ei voi mitata aikaa, vaan ajalla voidaan mitata tapahtumia. Aikaa ei ole olemassa omana erillisenä komponenttinaan, vaan keinotekoisena sovelluksena.


      • Anonyymi
        Anonyymi kirjoitti:

        Siis tarkoitatko, että aika todistaa itsensä olemassaolon? No ei nyt ihan kuitenkaan. Tapahtumilla ei voi mitata aikaa, vaan ajalla voidaan mitata tapahtumia. Aikaa ei ole olemassa omana erillisenä komponenttinaan, vaan keinotekoisena sovelluksena.

        Aika nimenomaan seuraa tapahtumista. Ilman tapahtumia sitä ei olisi.

        Liike ei ole absoluuttista. Jos liikut avaruudessa niin ilman jotakin vertailukohdetta et voi liikettäsi havaita. Sama koskee aikaa. Jos ei ole tapahtumia vertailukohtina niin et voi havaita ajan kuluvan.


      • Anonyymi
        Anonyymi kirjoitti:

        Siis tarkoitatko, että aika todistaa itsensä olemassaolon? No ei nyt ihan kuitenkaan. Tapahtumilla ei voi mitata aikaa, vaan ajalla voidaan mitata tapahtumia. Aikaa ei ole olemassa omana erillisenä komponenttinaan, vaan keinotekoisena sovelluksena.

        Aika ei todista itse itseään, mutta se paljastaa itsensä ihmisen kokemuksessa. Välitöntä kokemusta ajasta ei voi edes epäillä, niin kuin ei muitakaan välittömiä kokemuksia. Siten aika (tarkemmin ajan kuluminen) on otettava sellaisenaan tosiasiana.


      • Anonyymi
        Anonyymi kirjoitti:

        Aika ei todista itse itseään, mutta se paljastaa itsensä ihmisen kokemuksessa. Välitöntä kokemusta ajasta ei voi edes epäillä, niin kuin ei muitakaan välittömiä kokemuksia. Siten aika (tarkemmin ajan kuluminen) on otettava sellaisenaan tosiasiana.

        Ajattelet koko ajan 'ajan määritelmän' kautta, koska sinulla ei ole vaihtoehtoa, jonka voisit tiedostaa ja vaihtaa kokemusta.

        Jos mietit tarkemmin aivojen toimintaa: ihmisen kehittynyt muisti ja havainnointikyky mahdollistaa dynaamisen, selektiivisen ja pidemmän kokemuksen tapahtumista ja niiden kestosta. Tämä on se alkuperäinen ja primitiivinen ominaisuus, jolle ei ole aina ollut selitystä tai määritelmää, eikä edes taitoa siihen. Esim. talven kestoa ei voitu mitata ja oppia tietämään sen kestoa mitallisesti tai määrällisesti. Se oli vain primitiivinen ja epämääräinen kokemus kestosta. Kokemuksellisuus muuttuu, kun sovitetaan asia tiettyihin määriteltyihin tasavälisiin jaksoihin ja niiden nimettyihin yksiköihin ja niitä vastaaviin merkkeihin, sekä nimitetään tätä "ajaksi". Silloin syntyy yhteistuloksena "ajallinen mittaus", sekä siinä ohella kehittyy mieleen "aikakäsitys".

        Miksi aika kulkee? Se kokemus perustuu suoraan mittaamisen ominaisuuteen, jossa mitataan jonkin kestoa juoksevasti, eli esim. tiimalasissa hiekka liikkuu ohuen kaulan läpi pullon alaosaan. tai analogisessa kellossa, jossa osoittajat liikkuvat järjestyksessä numerosta toiseen... 1,2,3....

        Miksi aika kuluu? Se kokemus perustuu ajallisen mittaamisen kohdistuessa johonkin asiaan, jolla on rajallinen aika. Esim. ihmisen elämä, talven kesto, matka-aika jne. Näiden kohdalla aika kuluu, kunnes mittauksen kohteessa mittaus päättyy.

        Ajan "liike" ja "kuluminen" on siis ihan käytännössä syntyneitä kokemuksia, eikä niitä olisi syntynyt ilman ajan keksimistä, määrittelyä, sekä sen käyttötapoja.

        Aika on siis näkymätön itsetehty "layeri" mielessämme, jonka läpi katsomme ja koemme maailmaa, joka muuttuu ilman aikaa.


      • Anonyymi
        Anonyymi kirjoitti:

        Ajattelet koko ajan 'ajan määritelmän' kautta, koska sinulla ei ole vaihtoehtoa, jonka voisit tiedostaa ja vaihtaa kokemusta.

        Jos mietit tarkemmin aivojen toimintaa: ihmisen kehittynyt muisti ja havainnointikyky mahdollistaa dynaamisen, selektiivisen ja pidemmän kokemuksen tapahtumista ja niiden kestosta. Tämä on se alkuperäinen ja primitiivinen ominaisuus, jolle ei ole aina ollut selitystä tai määritelmää, eikä edes taitoa siihen. Esim. talven kestoa ei voitu mitata ja oppia tietämään sen kestoa mitallisesti tai määrällisesti. Se oli vain primitiivinen ja epämääräinen kokemus kestosta. Kokemuksellisuus muuttuu, kun sovitetaan asia tiettyihin määriteltyihin tasavälisiin jaksoihin ja niiden nimettyihin yksiköihin ja niitä vastaaviin merkkeihin, sekä nimitetään tätä "ajaksi". Silloin syntyy yhteistuloksena "ajallinen mittaus", sekä siinä ohella kehittyy mieleen "aikakäsitys".

        Miksi aika kulkee? Se kokemus perustuu suoraan mittaamisen ominaisuuteen, jossa mitataan jonkin kestoa juoksevasti, eli esim. tiimalasissa hiekka liikkuu ohuen kaulan läpi pullon alaosaan. tai analogisessa kellossa, jossa osoittajat liikkuvat järjestyksessä numerosta toiseen... 1,2,3....

        Miksi aika kuluu? Se kokemus perustuu ajallisen mittaamisen kohdistuessa johonkin asiaan, jolla on rajallinen aika. Esim. ihmisen elämä, talven kesto, matka-aika jne. Näiden kohdalla aika kuluu, kunnes mittauksen kohteessa mittaus päättyy.

        Ajan "liike" ja "kuluminen" on siis ihan käytännössä syntyneitä kokemuksia, eikä niitä olisi syntynyt ilman ajan keksimistä, määrittelyä, sekä sen käyttötapoja.

        Aika on siis näkymätön itsetehty "layeri" mielessämme, jonka läpi katsomme ja koemme maailmaa, joka muuttuu ilman aikaa.

        Olen erimieltä siitä, että kokemus ajasta muuttuisi, jos mittaamme aikaa tai esittämme sitä yksikköinä (sekunti tms.). En kutsuisi ajan kokemusta primitiiviseksi. Parempi ilmaisu on aito tai alkuperäinen, välitön kokemus.

        Sen sijaan toteamus siitä, että ajallinen mittaaminen ajanyksikköinä on toisenlainen tosiasia kuin välitön ajan kulun kokeminen. Se muodostuu silloin sosiaaliseksi ja yhteisesti jaettavaksi kokemukseksi ajasta. Sitä voidaan nimittää keinotekoiseksi ajan kokemukseksi johtuen juurikin näistä keksityistä yksiköistä.

        Sotket siis virheellisesti ajan välittömän kokemuksen ja sen sosiaalisen, keinotekoisen (mitatun) muodon keskenään.


      • Anonyymi
        Anonyymi kirjoitti:

        Olen erimieltä siitä, että kokemus ajasta muuttuisi, jos mittaamme aikaa tai esittämme sitä yksikköinä (sekunti tms.). En kutsuisi ajan kokemusta primitiiviseksi. Parempi ilmaisu on aito tai alkuperäinen, välitön kokemus.

        Sen sijaan toteamus siitä, että ajallinen mittaaminen ajanyksikköinä on toisenlainen tosiasia kuin välitön ajan kulun kokeminen. Se muodostuu silloin sosiaaliseksi ja yhteisesti jaettavaksi kokemukseksi ajasta. Sitä voidaan nimittää keinotekoiseksi ajan kokemukseksi johtuen juurikin näistä keksityistä yksiköistä.

        Sotket siis virheellisesti ajan välittömän kokemuksen ja sen sosiaalisen, keinotekoisen (mitatun) muodon keskenään.

        Ymmärsit hieman väärin. 'Primitiivinen tapahtumien keston hahmottaminen' tarkoittaa keston kokemista ilman järjestelmää, eli epämääräisenä, esim. ei synny ajallisten etäisyyksien kokemusta, esim. "vuosi sitten" tai "toisen maailmansodan aikana". Aikajärjestelmän avulla mieli oppii vähitellen tunnistamaan ajalliset jaksot, esim. minuutit, viikot, vuodet...

        Tapahtumien keston hahmottaminen on normaali ihmisaivojen ominaisuus ja se on se perusta, jonka avulla voitiin keksiä aikajärjestelmä. Ja se on ainoa mitä meillä on, yksi ainoa 'aika'. Eli ei ole olemassa kahta erillistä aikaa, kuten varsinainen aika ja keinotekoinen mitattu aika. Aika on yksi ainoa keinotekoinen järjestelmä, jota ihminen lähti luomaan ja kehittämään tuhansia vuosia sitten.

        Koko tämän kehityksen perusta on kehittynyt muisti, jonka avulla voi oppia ja kehittyä edelleen. Sen tuloksena on syntynyt monelaiset hyödylliset keksinnöt, kuten symbolit, kommunikaatio, työkalujen valmistus jne. Tässä kehityksessä aika on yksi niistä keksinnöistä, aivan kuten paino ja pituuskin. Nämä kolme keksintöä perustuu kaikki täysin ihmisen omiin kokemuksiin elämästä. Tapahtumien keston hahmottamisesta tuli aikajärjestelmä, painon ja painoerojen hahmottamisesta painojärjestelmä jne...


      • Anonyymi
        Anonyymi kirjoitti:

        Ymmärsit hieman väärin. 'Primitiivinen tapahtumien keston hahmottaminen' tarkoittaa keston kokemista ilman järjestelmää, eli epämääräisenä, esim. ei synny ajallisten etäisyyksien kokemusta, esim. "vuosi sitten" tai "toisen maailmansodan aikana". Aikajärjestelmän avulla mieli oppii vähitellen tunnistamaan ajalliset jaksot, esim. minuutit, viikot, vuodet...

        Tapahtumien keston hahmottaminen on normaali ihmisaivojen ominaisuus ja se on se perusta, jonka avulla voitiin keksiä aikajärjestelmä. Ja se on ainoa mitä meillä on, yksi ainoa 'aika'. Eli ei ole olemassa kahta erillistä aikaa, kuten varsinainen aika ja keinotekoinen mitattu aika. Aika on yksi ainoa keinotekoinen järjestelmä, jota ihminen lähti luomaan ja kehittämään tuhansia vuosia sitten.

        Koko tämän kehityksen perusta on kehittynyt muisti, jonka avulla voi oppia ja kehittyä edelleen. Sen tuloksena on syntynyt monelaiset hyödylliset keksinnöt, kuten symbolit, kommunikaatio, työkalujen valmistus jne. Tässä kehityksessä aika on yksi niistä keksinnöistä, aivan kuten paino ja pituuskin. Nämä kolme keksintöä perustuu kaikki täysin ihmisen omiin kokemuksiin elämästä. Tapahtumien keston hahmottamisesta tuli aikajärjestelmä, painon ja painoerojen hahmottamisesta painojärjestelmä jne...

        Milei oppii ajan yksiköiden hyödyntämisen, mutta sen lisäksi on olemassa aito ja välitön kokemus ajasta. Ei se mihinkään katoa, vaikka mittayksiköitä käytettäsiinkin ajan mittaamiseen. Etäisyyden kokemuskaan ei muutu, vaikka tunnemme metrit yms. pituuden mitat.

        Aika itsessään ei ole myöskään mikään keksintö. Ajan mittaaminen toki on vaatinut omat keksintönsä aurinkokellosta alkaen tähän päivään.

        Paino ei myöskään ole mikään keksintö, vaan sen voi havaita itse jokainen esim. punnitsemalla kädessään jonkin esineen painoa. Mittajärjestelmä painojen mittaukselle on jälleen eri asia.

        Jos ymmärsin mielestäsi jotakin väärin, niin olet selittänyt asiaa todella huonosti, koska ei ajan kokemuskessa ole mitään tuollaista mystistä "yhtä aikaa".


      • Anonyymi
        Anonyymi kirjoitti:

        Milei oppii ajan yksiköiden hyödyntämisen, mutta sen lisäksi on olemassa aito ja välitön kokemus ajasta. Ei se mihinkään katoa, vaikka mittayksiköitä käytettäsiinkin ajan mittaamiseen. Etäisyyden kokemuskaan ei muutu, vaikka tunnemme metrit yms. pituuden mitat.

        Aika itsessään ei ole myöskään mikään keksintö. Ajan mittaaminen toki on vaatinut omat keksintönsä aurinkokellosta alkaen tähän päivään.

        Paino ei myöskään ole mikään keksintö, vaan sen voi havaita itse jokainen esim. punnitsemalla kädessään jonkin esineen painoa. Mittajärjestelmä painojen mittaukselle on jälleen eri asia.

        Jos ymmärsin mielestäsi jotakin väärin, niin olet selittänyt asiaa todella huonosti, koska ei ajan kokemuskessa ole mitään tuollaista mystistä "yhtä aikaa".

        Mitään nykyään tunnettuja termejä, käsitteitä, järjestelmiä yms. ei ollut olemassakaan. Niiden olemassaolon väistämättömiä edellytyksiä on kehittyvä ja oppiva ajattelu ja tämän lähteenä on käytäntö. Kuulostaa hurjalta, että jonkin asian keksimisen jälkeen esitetään väite, ettei sitä olekaan keksitty, vaan löydetty.

        Ongelma ei ole niinkään mun selityksissäni, vaan siinä, ettet ensinnäkään hyväksy, että aika on ihmisen luoma keksintö. Yhtä hyvin voi väittää, että puhekin oli jo olemassa, joka piti vain löytää. Ja kirjaimet ja numerot. Ja matematiikka. Ja filosofia. Kun otat jonkin esineen käteen, tietenkin nyt tunnet "painon", käsitteen varsinaisessa merkityksessä, mutta 10000 vuotta sitten et olisi voinut ymmärtää tuntevasi "painoa", koska kyseistä termiä ja käsitettä merkitsemään kyseistä havaintoa ja kokemusta, ei ollut vielä keksitty. Aikaa et myöskään voi kokea, ennen kuin keksit kyseisen termin ja käsitteen merkitsemään tapahtumien keston havaitsemista ja kokemista. Ja siitä on kyse: miten koettiin tapahtumien kesto, ennen kuin sitä ryhdyttiin selittämään ja määrittelemään keksinnöllä nimeltä "aika". Ja kun tämän ymmärtää, niin silloin ymmärtää myös sen, miten mieli on ohjelmoitu kokemaan "aika" itsenäisenä konkreettisena komponenttina, joka edelleen huijaa jopa meritoituneen tiedemiehen uskomaan, että aika on ollut olemassa alusta asti, joka piti vain löytää. Todellisuudessa kuitenkin kyse on ihmisaivojen ominaisuudesta, jota merkitsemään keksittiin ja kehitettiin termi, käsite, määritelmä, käyttötavat jne. Näin voimme todeta, että aika on ihmisen keksintö, ei universumissa oleva itsenäinen komponentti tms.


      • Anonyymi
        Anonyymi kirjoitti:

        Mitään nykyään tunnettuja termejä, käsitteitä, järjestelmiä yms. ei ollut olemassakaan. Niiden olemassaolon väistämättömiä edellytyksiä on kehittyvä ja oppiva ajattelu ja tämän lähteenä on käytäntö. Kuulostaa hurjalta, että jonkin asian keksimisen jälkeen esitetään väite, ettei sitä olekaan keksitty, vaan löydetty.

        Ongelma ei ole niinkään mun selityksissäni, vaan siinä, ettet ensinnäkään hyväksy, että aika on ihmisen luoma keksintö. Yhtä hyvin voi väittää, että puhekin oli jo olemassa, joka piti vain löytää. Ja kirjaimet ja numerot. Ja matematiikka. Ja filosofia. Kun otat jonkin esineen käteen, tietenkin nyt tunnet "painon", käsitteen varsinaisessa merkityksessä, mutta 10000 vuotta sitten et olisi voinut ymmärtää tuntevasi "painoa", koska kyseistä termiä ja käsitettä merkitsemään kyseistä havaintoa ja kokemusta, ei ollut vielä keksitty. Aikaa et myöskään voi kokea, ennen kuin keksit kyseisen termin ja käsitteen merkitsemään tapahtumien keston havaitsemista ja kokemista. Ja siitä on kyse: miten koettiin tapahtumien kesto, ennen kuin sitä ryhdyttiin selittämään ja määrittelemään keksinnöllä nimeltä "aika". Ja kun tämän ymmärtää, niin silloin ymmärtää myös sen, miten mieli on ohjelmoitu kokemaan "aika" itsenäisenä konkreettisena komponenttina, joka edelleen huijaa jopa meritoituneen tiedemiehen uskomaan, että aika on ollut olemassa alusta asti, joka piti vain löytää. Todellisuudessa kuitenkin kyse on ihmisaivojen ominaisuudesta, jota merkitsemään keksittiin ja kehitettiin termi, käsite, määritelmä, käyttötavat jne. Näin voimme todeta, että aika on ihmisen keksintö, ei universumissa oleva itsenäinen komponentti tms.

        Olet täydellisesti ymmärtänyt väärin kokemuksen ja kielen suhteet. Erityisesti väärin oli väittää, ettei kieltä olisi keksitty. Kielen käyttö on ollut ihmiskunnan tärkein keksintö.

        Se ei kuitenkaan tarkoita, että välitön kokemus esineiden painosta olisi puuttunut, koska sille asialle ei ole käsitettä "paino".

        Tottakai painon voi tuntea ilman mitään käsitettä painosta, koska se on välitön kokemus ihan kuin löisit varpaasi kiveen. Silloin sattuu ja kovasti! Esineen painon tuntee myös, vaikka kivuliaaksi sen kokee vasta, kun paino olisi niin suuri, että esine putoaa varpaillesi.

        Ajan kokeminen on myös ihmsien välitön kokemus eikä siihen tarvita kelloa eikä edes "ajan" käsitettä.

        Aika on universumin ominaisuus ja se käsitetään aika-avaruuteen kuuluvana kuten Einsteinin suhteellisuusteoriat sen selittävät.


      • Anonyymi
        Anonyymi kirjoitti:

        Olet täydellisesti ymmärtänyt väärin kokemuksen ja kielen suhteet. Erityisesti väärin oli väittää, ettei kieltä olisi keksitty. Kielen käyttö on ollut ihmiskunnan tärkein keksintö.

        Se ei kuitenkaan tarkoita, että välitön kokemus esineiden painosta olisi puuttunut, koska sille asialle ei ole käsitettä "paino".

        Tottakai painon voi tuntea ilman mitään käsitettä painosta, koska se on välitön kokemus ihan kuin löisit varpaasi kiveen. Silloin sattuu ja kovasti! Esineen painon tuntee myös, vaikka kivuliaaksi sen kokee vasta, kun paino olisi niin suuri, että esine putoaa varpaillesi.

        Ajan kokeminen on myös ihmsien välitön kokemus eikä siihen tarvita kelloa eikä edes "ajan" käsitettä.

        Aika on universumin ominaisuus ja se käsitetään aika-avaruuteen kuuluvana kuten Einsteinin suhteellisuusteoriat sen selittävät.

        "Yhtä hyvin voi väittää, että puhekin oli jo olemassa, joka piti vain löytää."

        Luetun ymmärtäminen?

        Keskustelu on joka tapauksessa hedelmätöntä, koska et ymmärrä ihmisen kehittymisen seurauksia: oppiminen, tiedon luominen, käsitteiden ja termien luominen jne. ja miten tämä vaikuttaa ajatteluun ja kokemiseen. "Aika" on luotua tietoa, luotu käsite ja termi, luotu järjestelmä, jolle on luotu merkitys, ja joka on alkeellisen, määrittelemättömän, aivojen ominaisuuden kehittynyt verbaalinen vastine. Samoin "paino" ja "pituus". Mitään tietoa ei ole ennalta olemassa, vaan se luodaan itse. Aika ole koskaan ollut universumin ominaisuus, vaan ihmisen luoma keksintö, jota sovelletaan universumiin, sen lisäksi, että sitä sovelletaan arkielämäänkin. Suhtiksessa aika on myös sovelluksena, mutta silti suhtista väärinkäytetään "todistamaan" ajan olemassaolo itsenäisenä komponenttina. Ei se sitä todista, vaan sitä käytetään osana suhtista.

        Aikaa ei ole olemassa itsenäisenä komponenttina, vaan luotuna tietona.


      • Anonyymi
        Anonyymi kirjoitti:

        "Yhtä hyvin voi väittää, että puhekin oli jo olemassa, joka piti vain löytää."

        Luetun ymmärtäminen?

        Keskustelu on joka tapauksessa hedelmätöntä, koska et ymmärrä ihmisen kehittymisen seurauksia: oppiminen, tiedon luominen, käsitteiden ja termien luominen jne. ja miten tämä vaikuttaa ajatteluun ja kokemiseen. "Aika" on luotua tietoa, luotu käsite ja termi, luotu järjestelmä, jolle on luotu merkitys, ja joka on alkeellisen, määrittelemättömän, aivojen ominaisuuden kehittynyt verbaalinen vastine. Samoin "paino" ja "pituus". Mitään tietoa ei ole ennalta olemassa, vaan se luodaan itse. Aika ole koskaan ollut universumin ominaisuus, vaan ihmisen luoma keksintö, jota sovelletaan universumiin, sen lisäksi, että sitä sovelletaan arkielämäänkin. Suhtiksessa aika on myös sovelluksena, mutta silti suhtista väärinkäytetään "todistamaan" ajan olemassaolo itsenäisenä komponenttina. Ei se sitä todista, vaan sitä käytetään osana suhtista.

        Aikaa ei ole olemassa itsenäisenä komponenttina, vaan luotuna tietona.

        Ymmärrän hyvin, että aikaa voi mitata ja sen käsitteellinen ymmärtäminen on erilaista kuin ajan välitö kokeminen. Tämä kokemus ei ole tietoa vaan se on kokemus, jota ei voi tosiasiassa edes epäillä - toisin kuin tietoa, jota voidaan aina epäillä, eli tieto on usein epävarmaa.

        Suhteellisuusteoriat eivät tietenkän todista mitään, vaan auttavat ymmärtämään ajan nimenomaan käsitteellisenä eli juuri siinä muodossa kuin sinä jankkaat.

        Tiedämme kuitenkin, että suhtiksen aikadilataation seurauksena avaruusmatkalla ollut astronautti voi palata täysin eri ikäisenä kuin kaksoisveljensä, joka jää Maahan odottelemaan veljensä paluuta. Tämä on hyvin konkreettinen seuraus ajan kulumisesta. Se tapahtuu, vaikka kumpikaan heistä ei olisi koskaan kuullutkaan suhtiksesta.

        PS. Ja ymmärsin, että teit vertauksen puheeseen keksintönä. Korostin vain, että se ei ole oikea vertailukohde ajan kulumiseen, sillä kieli on todellakin ihmisen keinotekoinen keksintö, eikä sitä ole olemassa missään luonnossa eikä luonnollisessa kokemuksessa. Kieli, jos mikä, on käsitteellistä.


    • Anonyymi

      Ajalla ei ole mitään merkitystä. Suhteellisuusteoriasakaan ei puhuta ajasta mitään. Ihmisen luoma keinotekoinen tuote.

      • Anonyymi

        Miksi sitten suhtiksen kaavoissa on joka paikassa mukana aika, jotta saataisiin laskettua tulokset oikein? Olet tuossa nyt kyllä erehtynyt.

        Katso Lorentzin muunnokset.


      • Anonyymi
        Anonyymi kirjoitti:

        Miksi sitten suhtiksen kaavoissa on joka paikassa mukana aika, jotta saataisiin laskettua tulokset oikein? Olet tuossa nyt kyllä erehtynyt.

        Katso Lorentzin muunnokset.

        Alkeellisissa fysiikan teorioissa ja yhtälöissä on aika, joka on parametri t. Vähän kehittyneemmissä on aikaoperaattori joka voi olla myös diskreetti ja antaa hiukkasen aikajakauman, ja kattavimmissa teorioissa ei ole aikaa ollenkaan. Tällöin katsotaan, että aika on kvanttilomittumisesta johtuva emergentti ilmiö, mutta kaavoihin sitä ei saa lisättyä.
        https://fi.wikipedia.org/wiki/Lomittuminen


    • Anonyymi

      Maailmankaikkeudessa ei ole kovin montaa tapahtumaa/ilmiötä jolla ei olisi alkua ja loppua tai muutosta/syklejä. Ehkä niitä harvoja on kun siirrytään ajasta ikuisuuteen.

    • Anonyymi

      "Jos väitän että aikaa ei ole olemassa, miten todistat väitteeni vääräksi?"

      Todistan väitteesi vääräksi koska et pysty todistamaan väitettäsi oikeaksi.

      • Anonyymi

        No todista.


    • Anonyymi

      jännä juttu sinänsä koska aikahan on muuttuva eli suhteellinen käsite eli jokaisella ihmisellä on mielessään oma aikansa, niin miten sellaista voi mitenkään sovaeltaa mihinkään?

      Aitpkemlom mpåeis åot'oso ,otata airomgpm s'tweost' ,pmtalp vaöp,oööoselimtoa s'teess' tao kptaom---

    • Anonyymi

      Aika on katsojan silmässä, jos on paljon erähmää silmässä aika coi muurtruakin.

      Törmäsin itse kun 3d-juttuja, erikoiseen juttuun piinneliöjuuri ja miten käytetään, hweti kyllä tajusin että ei 2-tason ympyreöitä :D

    • Anonyymi

      koska aika on liian iso jotta olisit oikeassa

    • Anonyymi

      koska olet hiekan jyvä meren rannalla

      kuin

      roska avaruudessa

    • Anonyymi

      koska et kerkeä laskea hiekkajyvien määriä hiekkakellon aikana

    • Anonyymi

      koska olet filosofi ja saat väittää kaikkea vapaasti

    • Anonyymi

      koska filosofeilla on aikaa väitellä

    • Anonyymi

      koska filosofeilla on aikaa joutilaaseen väittelyyn, niin hän yrittää väittää, että aikaa ei ole olemassa, koska aika on iätön

      mutta pitää sekunttimittarilla oikein mitata, että koska filosofin aika loppuu

      • Anonyymi

        No nyt on tajunnanvirtaa, kuin yleisessä pisuaarissa XD Koitahan Tero vähän eheytyä ja keksiä jotain järkevämpää.


    • Anonyymi

      filosofin aikaa loppuu silloin kun filosofi ei jaksa enää ajatella

      silloin on pakko luovuttaa ja sanoa, että aika on olemassa jälleen

      siis

      filosofi voi pysäyttää ajan ja kokea että aikaa ei ole olemassa

      mutta filosofi ei osaa pitää ajan aloillaan liian pitkään

      siis

      keho biologia on aika

      aivot ovat liha

      liha on kello

      kello ei ole rannekello

      kuollut liha on huono kello

      elävät aivot on aika

      siis

      FILOSOFI ON ITSE AIKA

      mutta koska filosofi sanoo, että aikaa ei ole olemassa
      se tarkoittaa, että FILOSOFI EI TAHDO OLLA AIKA
      filosofi ei tahdo kulua, vanheta, edistyä, juota, tehdä ym.

      filosofi yrittää pysäyttää ajan järjellään
      mikä on mielenkiintoinen tapahtuma
      josta minä itsekin pidän
      koska se tuo endorfiini tunteita
      ja on kuin huume konsanaan

      järjellä on kuitenkin maailmanlaajuisestikin suurempi merkitys
      ja jokainen ajan pysäytys on hyvä harjoitus
      että miljoonan ajan pysäytyksen jälkeen
      filosofi osaa pysäyttää ajan sellaisella tavalla
      josta maailmankaikkeus voi yllättyä

      kaikella on aikansa tai hintansa

    • Anonyymi

      Aikahan on vain jatkumo. Jos ei mitään ole, eikä tapahdu ei aikaakaan ole.

      • Anonyymi

        Aika saattaa olla myös kvantittunut jolloin se vastaa luonnollisten lukujen joukkoa, eikä ole jatkumo.


      • Anonyymi
        Anonyymi kirjoitti:

        Aika saattaa olla myös kvantittunut jolloin se vastaa luonnollisten lukujen joukkoa, eikä ole jatkumo.

        Hä??! XD


    • Anonyymi

      Ps. 112: 5–6 armeliaisuudella on aikansa.

    • Anonyymi

      Aika on subjektiivinen kokemus, joka on ns. synkronoitu kellon tahtiin. Koska kello on todellinen, siinä nähdyn ajankin koetaan olevan kellon kautta todellinen. Näin koetaan ajan liikkuvan, etenevän, kuluvan jne. Ja näin olemme ohjelmoineet/manipuloineet mielemme ajantajulla.

      • Anonyymi

        Olet päätellyt asiat nurinkurisesti.

        Eläimet ennen ihmistä olivat lukkiutuneet vuorokausirytmiin.
        Kun ihminen ilmestyi maailmaan, hän lukkiutui samaan rytmiin.

        Ihmisen aivoissa on "kello" joka tahdistuu vuorokausirytmiin valon
        vaikutuksesta.
        Vuorokausirytmi olikin ihmisten mielestä aika, joten kellot tehtiin
        mittaamaan valoisaa aikaa. Ensimmäiset egyptiläisten aurinkokellot
        mittasivat tietysti paikallista aurinkoaikaa. Egyptiläisten mielestä yöllä
        ei ollut aikaa.

        Nykyiset kellot ja ajanmittausjärjestelmä jatkaa tätä samaa perinnettä,
        kellot on tahdistettu 24 tunnin jaksoon.

        Mutta aika ei ole kiinni siitä mitä tajuamme kellon kautta, vaan kellosta
        riippumattomat tapahtumat ovat se missä aika luuraa,

        Ihmisillä on aikakäsite, mutta se ei ole aika.
        Ihmisillä on myös lehmäkäsite, mutta se ei ole lehmä.

        Koska meillä on lehmäkäsite tiedämme mitä lehmästä vai saada.

        Ajantaju on vanha evoluution keksintö.


      • Anonyymi
        Anonyymi kirjoitti:

        Olet päätellyt asiat nurinkurisesti.

        Eläimet ennen ihmistä olivat lukkiutuneet vuorokausirytmiin.
        Kun ihminen ilmestyi maailmaan, hän lukkiutui samaan rytmiin.

        Ihmisen aivoissa on "kello" joka tahdistuu vuorokausirytmiin valon
        vaikutuksesta.
        Vuorokausirytmi olikin ihmisten mielestä aika, joten kellot tehtiin
        mittaamaan valoisaa aikaa. Ensimmäiset egyptiläisten aurinkokellot
        mittasivat tietysti paikallista aurinkoaikaa. Egyptiläisten mielestä yöllä
        ei ollut aikaa.

        Nykyiset kellot ja ajanmittausjärjestelmä jatkaa tätä samaa perinnettä,
        kellot on tahdistettu 24 tunnin jaksoon.

        Mutta aika ei ole kiinni siitä mitä tajuamme kellon kautta, vaan kellosta
        riippumattomat tapahtumat ovat se missä aika luuraa,

        Ihmisillä on aikakäsite, mutta se ei ole aika.
        Ihmisillä on myös lehmäkäsite, mutta se ei ole lehmä.

        Koska meillä on lehmäkäsite tiedämme mitä lehmästä vai saada.

        Ajantaju on vanha evoluution keksintö.

        "Egyptiläisten mielestä yöllä ei ollut aikaa."

        Muinaisilla egyptiläisillä ja babylonialaisilla oli tiimalaseja, joilla aikaa pystyi mittaamaan yölläkin.


      • Anonyymi
        Anonyymi kirjoitti:

        "Egyptiläisten mielestä yöllä ei ollut aikaa."

        Muinaisilla egyptiläisillä ja babylonialaisilla oli tiimalaseja, joilla aikaa pystyi mittaamaan yölläkin.

        Kreikkalaisilla, roomalaisilla ja egyptiläisillä oli myös vesikelloja eli klepsydroita, joissa ajan mittaaminen perustuu veden virtaamiseen. Vanhimmat klepsydrat ovat 4000 vuotta vanhoja.


      • Anonyymi
        Anonyymi kirjoitti:

        Kreikkalaisilla, roomalaisilla ja egyptiläisillä oli myös vesikelloja eli klepsydroita, joissa ajan mittaaminen perustuu veden virtaamiseen. Vanhimmat klepsydrat ovat 4000 vuotta vanhoja.

        Joku egyptiläinen siis keksi myöhemmin että myös yöllä on aika.


      • Anonyymi
        Anonyymi kirjoitti:

        Joku egyptiläinen siis keksi myöhemmin että myös yöllä on aika.

        Silloinen alkeellinen ajattelutapa oli vain sitä, ettei ilman auringon varjoa voitu mitata aikaa. Ei silloin ajateltu, että "päivällä on aika", vaan "päivää voi mitata ajannäyttimellä". Kun ajan mittaaminen vakiintui ja kehittyi, sitä voitiin soveltaa myös yöhön. Ja lopulta kehityksen myötä soveltamistavat monipuolistui, kuin myös mittausmenetelmät. Ja missä ollaan nyt... aika nähdään osana koko universumia ja se löytyy myös teorioista.


      • Anonyymi
        Anonyymi kirjoitti:

        Silloinen alkeellinen ajattelutapa oli vain sitä, ettei ilman auringon varjoa voitu mitata aikaa. Ei silloin ajateltu, että "päivällä on aika", vaan "päivää voi mitata ajannäyttimellä". Kun ajan mittaaminen vakiintui ja kehittyi, sitä voitiin soveltaa myös yöhön. Ja lopulta kehityksen myötä soveltamistavat monipuolistui, kuin myös mittausmenetelmät. Ja missä ollaan nyt... aika nähdään osana koko universumia ja se löytyy myös teorioista.

        No nyt kyllä - aurinkokellon lisäksi käytössä oli vesikello, jolla pystyttiin mittaamaan yleensä hiukan lyhyempiä tapahtumia hiukan tarkemmin. Tiedän ainakin, että vesikellolla saatettiin määritellä rukoushetkien ajankohtia ja käytössä oli ainakin 6h valuvia kelloja.


    • Anonyymi

      Ensiksi pitäisi tarkentaa kysymystä. Esimerkiksi: Mitä tarkoitetaan ajalla?

      Näyttäisi olevan vain prosessien tapahtumisjärjestyksiä, saman"-aikaisuuuksia", kestoja (duraatioita) ja välejä (intervalleja) sekä näistä konstruoituja "ajan" empiirisiä mittoja, kalentereita yms. ja lopulta fiktiivinen - tai olkoon: abstrakti "ajan" dimensio.

      Kalentereita voi selailla ees-sun-taas kuten videoitakin, ja taivaalta kuvailla gigavuosien takaisia galakseja, mutta olen vakuuttunut, että aikamatkustus (että siis elelisin syntymäpäiväni uudelleen biologisesti tms.) jää ja tulee jäämään ns. yliluonnolliseen todellisuuteen.

      • Anonyymi

        "olen vakuuttunut, että aikamatkustus (että siis elelisin syntymäpäiväni uudelleen biologisesti tms.) jää ja tulee jäämään ns. yliluonnolliseen todellisuuteen."

        Vaikka aikamatkustus olisi todellista ja tietoisuuden jatkumo olisi juuri 10 sekuntia sitten matkustanut 10 vuotta ajassa taaksepäin siihen hetkeen kun aloit lukea tätä ketjua, ei sitä pysty mistään huomaamaan. Aivoissa ei ole muistijälkiä 10 vuoden päästä tapahtuvista asioista, vaan tietoisuuden näyttämöllä on vain tämänhetkiset asiat ja aivoissa olevat muistikuvat.


      • Anonyymi
        Anonyymi kirjoitti:

        "olen vakuuttunut, että aikamatkustus (että siis elelisin syntymäpäiväni uudelleen biologisesti tms.) jää ja tulee jäämään ns. yliluonnolliseen todellisuuteen."

        Vaikka aikamatkustus olisi todellista ja tietoisuuden jatkumo olisi juuri 10 sekuntia sitten matkustanut 10 vuotta ajassa taaksepäin siihen hetkeen kun aloit lukea tätä ketjua, ei sitä pysty mistään huomaamaan. Aivoissa ei ole muistijälkiä 10 vuoden päästä tapahtuvista asioista, vaan tietoisuuden näyttämöllä on vain tämänhetkiset asiat ja aivoissa olevat muistikuvat.

        Tarkoitatko, että aikamatkustuksen sivuoireena olisi matkatun ajan mittainen amnesia? Eli matkaaja ei voi viedä tämänhetkisiä kokemuksia muistoina 10 vuoden päähän menneisyyteen?


      • Anonyymi

        "Ensiksi pitäisi tarkentaa kysymystä. Esimerkiksi: Mitä tarkoitetaan ajalla?"

        Sinulla oli hyvä luettelo perustekijöistä, mutta miksi "vain", iso kuva
        jää muodostumatta.
        Luettelosi koostui tapahtumien sisäisistä suhteista ja tapahtumien välisistä
        suhteista, jotakin tärkeää tuli siis näkyville.

        Fyysikkoja siteeraten, todellisuus koostuu tapahtumista, ei objekteista.
        Me elämme tapahtumien kokonaisuudessa, minne tulee koko myös uusia
        tapahtumia, meidän kehomme koostuu tapahtumista ja tapahtumaketjuista,
        tapahtumat tappavat meidät ja vaikuttavat millaiseksi elämämme muodostuu.

        Tuo kysymys "Mitä tarkoitetaan ajalla?" pitäisi tarkentaa muotoon,
        "Mitä tarkoitetaan aikatermillä?", koska termillä on merkitys, se tuo mukaan
        käsitteen.

        Vielä tarkempi kysymys olisi siis "Mikä on ajan käsite?".
        Koska kysymys olisi ilmiötason ajasta, eikä fyysikoiden etsimästä vielä
        tuntemattomasta ajasta, ajan käsite kuvaisi tapahtumien suhteita, koska
        ne olisivat se olennainen tekijä ilmiötasolla.

        Työmääritelmä voisi olla "Ajalla tarkoitetaan tapahtumien temporaalisten
        primitiivisuhteiden muodostamaa kokonaisuutta".

        Tuossa "temporaaliset primitiivisuhteet" tarkoittaa ajallisluonteisia, kuten ennen,
        jälkeen, oli, nykyisyys, samanaikainen yms. suhteita.

        Ei olisi siis aikaobjekteja, kuten hetki, vain suhteita.
        Ajan käsite olisi siis abstraktisempi kuin primitiivitermit, tuo korkeampi
        abstraktiivisuus tekee "ajasta" vaikeammin hahmotettavan, mikä näkynee
        tämänkin palstan kommenteissa.

        R.


      • Anonyymi
        Anonyymi kirjoitti:

        "olen vakuuttunut, että aikamatkustus (että siis elelisin syntymäpäiväni uudelleen biologisesti tms.) jää ja tulee jäämään ns. yliluonnolliseen todellisuuteen."

        Vaikka aikamatkustus olisi todellista ja tietoisuuden jatkumo olisi juuri 10 sekuntia sitten matkustanut 10 vuotta ajassa taaksepäin siihen hetkeen kun aloit lukea tätä ketjua, ei sitä pysty mistään huomaamaan. Aivoissa ei ole muistijälkiä 10 vuoden päästä tapahtuvista asioista, vaan tietoisuuden näyttämöllä on vain tämänhetkiset asiat ja aivoissa olevat muistikuvat.

        Oivallinen päätelmä. Jos olisi kysymys oikeasta aikamatkustuksesta, niin omaan syntymähetkeen matkustaminen tarkoittaisi, että et osaa edes puhua. Kukaan ei syntymähetkellä nimittäin sitä osaa.

        Aikamatkukstuksen fiktiivisissä tarinoissa mennyt aika itse asiassa sekoitetaan aikamatkustajan lähtöajankohdan aikaan. Tämä ei aidossa aikamatkustuksessa pitäisi olla mahdollista. Ei siis voi olla matkaajaa omassa ajassaan niin, että hän otti lähtöhetken ajan mukaansa ja liikkuu sitten menneessä ajassa.


      • Anonyymi
        Anonyymi kirjoitti:

        Oivallinen päätelmä. Jos olisi kysymys oikeasta aikamatkustuksesta, niin omaan syntymähetkeen matkustaminen tarkoittaisi, että et osaa edes puhua. Kukaan ei syntymähetkellä nimittäin sitä osaa.

        Aikamatkukstuksen fiktiivisissä tarinoissa mennyt aika itse asiassa sekoitetaan aikamatkustajan lähtöajankohdan aikaan. Tämä ei aidossa aikamatkustuksessa pitäisi olla mahdollista. Ei siis voi olla matkaajaa omassa ajassaan niin, että hän otti lähtöhetken ajan mukaansa ja liikkuu sitten menneessä ajassa.

        Aito aikamatkustus? :D Eli tuo on vain yksi fiktiivisen aikamatkustuksen selitystapa.

        Eli tämä "aito aikamatkustus" tarkoittaisi esittämässäs ideassa sitä, että matkaaja palaisi täsmälleen samaan tilaan, asemaan, muotoon, järjestykseen jne. mennessään ajassa taaksepäin.

        Miten ns. aidossa aikamatkustamisessa voidaan esim. palata aikaan ennen ajassa liikkumiseen tarvittavan menetelmän keksimistä? Eikä matkaajan näkökulmasta edes tapahtuisi aikamatkustusta, koska hän ei muistaisi sitä. Ja jos ei, kenen näkökulmasta sitten?

        Tämä jokatapauksessa sotii koko fiktiivisen aikamatkustamisen ideaa vastaan.


      • Anonyymi
        Anonyymi kirjoitti:

        Aito aikamatkustus? :D Eli tuo on vain yksi fiktiivisen aikamatkustuksen selitystapa.

        Eli tämä "aito aikamatkustus" tarkoittaisi esittämässäs ideassa sitä, että matkaaja palaisi täsmälleen samaan tilaan, asemaan, muotoon, järjestykseen jne. mennessään ajassa taaksepäin.

        Miten ns. aidossa aikamatkustamisessa voidaan esim. palata aikaan ennen ajassa liikkumiseen tarvittavan menetelmän keksimistä? Eikä matkaajan näkökulmasta edes tapahtuisi aikamatkustusta, koska hän ei muistaisi sitä. Ja jos ei, kenen näkökulmasta sitten?

        Tämä jokatapauksessa sotii koko fiktiivisen aikamatkustamisen ideaa vastaan.

        Puutun teidän keskusteluun...

        Menneisyyteen voi aika matkustaa 2 tavalla:

        Kelaamalla aikaa takaisinpäin, joilloin itse nuorentuu.

        Tai hyppäämällä menneisyyteen, jolloin itse pysyy samanikäisenä.

        :-)


      • Anonyymi

        "Ensiksi pitäisi tarkentaa kysymystä. Esimerkiksi: Mitä tarkoitetaan ajalla?"

        Vastaus kysymyksiin, mitä aika on, missä aika on, miten aika on jne., on tässä ihan lähellä, korviemme välissä ja mikä mielenkiintoisinta, se on ainoa paikka, jossa sillä voi olla minkäänlaisia mahdollisuuksia esiintyä. Ajan rakenne on 100% biologinen: mitä aika on, sen määritelmä, sovellustavat jne... tämä kaikki on yksinomaan aivojen luomaa. Vai asettuiko aika aivoihin luun läpi ja samalla kertoi itsestään, mitä hän on, miten hän on jne.? Jotenkin tavanomaisissa näkemyksissä ajasta saattaa ajoittain lehahtaa hieman samankaltainen tunkkainen odööri kuin uskomuksilla aaveisiin.

        Koko perusta ajalle ja aikakäsitykselle on biologian, fysiikan ja luovuuden fuusio, jossa biologia edustaa aistien ja aivojen toimintaa, fysiikka luonnon tapahtumia ja luovuus oppivaa uuden tuottamista. Ts. kun aika keksittiin ja sen soveltaminen osaksi elämää vakiintui, tietenkin sen seurauksena on "AIKAkäsitys". Tätä voisi havainnollistaa toisella termillä: mitä on "kipu"? Voitko sanoa tuntevasi "kipua", jos et tunne termiä ja käsitettä "kipu"? Aivan, et voi. Kokemuksesi on siis alkukantainen epämiellyttävä, pelottava tms., johon sinulla ei ole mitään verbaalista vastinetta. Näin toimii myös käsite "aika". Et voi kokea aikaa, et kuluttaa aikaa jne., eikä sinulla ole ajantajua, ennen kuin tunnet käsitteen "aika". On siis ymmärrettävä käsitteellisen ajattelun vaikutukset koettuun.

        Kun aikaa etsitään kapeakatseisesti vain kvanttifysiikkasta, BB:stä yms., ei koskaan voida ymmärtää ajan todellista luonnetta ja sen syntytapaa. Debattikin on ikuinen.


      • Anonyymi
        Anonyymi kirjoitti:

        Tarkoitatko, että aikamatkustuksen sivuoireena olisi matkatun ajan mittainen amnesia? Eli matkaaja ei voi viedä tämänhetkisiä kokemuksia muistoina 10 vuoden päähän menneisyyteen?

        Muistot eivät siirry menneisyyteen, ainoastaan ihmisen tietoisuus siirtyy, jolloin voi elää esim. syntymäpäivänsä monta kertaa uudelleen.
        Jos muistotkin siirtyisivät, silloin kyseessä ei olisi aikamatkustus, vaan hyppy rinnakkaistodellisuuteen, joka poikkeaa tämän todellisuuden menneisyydestä.


      • Anonyymi
        Anonyymi kirjoitti:

        Muistot eivät siirry menneisyyteen, ainoastaan ihmisen tietoisuus siirtyy, jolloin voi elää esim. syntymäpäivänsä monta kertaa uudelleen.
        Jos muistotkin siirtyisivät, silloin kyseessä ei olisi aikamatkustus, vaan hyppy rinnakkaistodellisuuteen, joka poikkeaa tämän todellisuuden menneisyydestä.

        Kuulostaa ristiriitaiselta. Eli aikamatkustus on mahdollista vain luoda takaisinpaluu, jossa jokainen yksittäinen osa tekee täsmälleen aiemmat liikkeet takaperin? Miten tämä rajoitetaan matkaavan yksilön kohdalle, että vain hänen osansa ja aikansa liikuu takaperin? Luodaanko tässä jokin kupla, jonka sisällä fyysiset muutokset tapahtuu, vai näkeekö muut ihmiset ympärillä, että matkaaja vain yhtäkkiä alkaa liikkumaan, puhumaan yms. takaperin ja matkaaja havaitsee kaiken kulkevan takaperin? Vai tapahtuuko matkustus silmänräpäyksessä, että matkaaja vain katoaa ja omasta näkökulmastaan on juuri putkahtamassa äidin kohdusta ulos?

        Miksi muistot ei siirry menneisyyteen? Mikä poistaa ne? Jos siirtyy ajassa taaksepäin muistot mukana, miksi se ei olisi matkustamista tässä todellisuudessa?


      • Anonyymi
        Anonyymi kirjoitti:

        Kuulostaa ristiriitaiselta. Eli aikamatkustus on mahdollista vain luoda takaisinpaluu, jossa jokainen yksittäinen osa tekee täsmälleen aiemmat liikkeet takaperin? Miten tämä rajoitetaan matkaavan yksilön kohdalle, että vain hänen osansa ja aikansa liikuu takaperin? Luodaanko tässä jokin kupla, jonka sisällä fyysiset muutokset tapahtuu, vai näkeekö muut ihmiset ympärillä, että matkaaja vain yhtäkkiä alkaa liikkumaan, puhumaan yms. takaperin ja matkaaja havaitsee kaiken kulkevan takaperin? Vai tapahtuuko matkustus silmänräpäyksessä, että matkaaja vain katoaa ja omasta näkökulmastaan on juuri putkahtamassa äidin kohdusta ulos?

        Miksi muistot ei siirry menneisyyteen? Mikä poistaa ne? Jos siirtyy ajassa taaksepäin muistot mukana, miksi se ei olisi matkustamista tässä todellisuudessa?

        "Eli aikamatkustus on mahdollista vain luoda takaisinpaluu, jossa jokainen yksittäinen osa tekee täsmälleen aiemmat liikkeet takaperin? Miten tämä rajoitetaan matkaavan yksilön kohdalle, että vain hänen osansa ja aikansa liikuu takaperin?"

        Ennemminkin kyse on tunneloitumisen kaltaisesta ilmiöstä. Samalla tavoin kuin hiukkaset eivät sijaitse tarkasti yhdessä ajanhetkessä, myös tietoisuuden ajanhetki noudattaa aaltomaista jakaumaa joka voidaan laskea aikaoperaattorilla. Pienellä todennäköisyydellä tietoisuus voi tunneloitua pitkänkin matkan ajassa eteenpäin tai taaksepäin, ja tunneloitumisen jälkeen tietoisuuden jatkumo jatkuu vaikka kauempanakin menneisyydessä tai tulevaisuudessa. Subjektiivisesti tunneloitumista ei huomaa mistään. Menneeseen ajanhetkeen joutuessaan käytettävissä on vain menneen ajanhetken muistikuvat.


      • Anonyymi
        Anonyymi kirjoitti:

        Puutun teidän keskusteluun...

        Menneisyyteen voi aika matkustaa 2 tavalla:

        Kelaamalla aikaa takaisinpäin, joilloin itse nuorentuu.

        Tai hyppäämällä menneisyyteen, jolloin itse pysyy samanikäisenä.

        :-)

        "hyppäämällä menneisyyteen, jolloin itse pysyy samanikäisenä."

        Niin kuin selitin aidosta aikamatkailusta, tässä hypyssä sekoittuu kaksi aikatasoa toisiinsa. Mitä aikamatkustusta sellainen on?

        Siinä nykyhetken ihminen sellaisenaan siinä nykyhetkessä siirtyy aika mukanaan menneisyyteen, jossa on toinen aikataso. Miten nämä aikatasot ovat vuorovaikutuksessa toisiinsa? Kysymyksiä herää valtava määrä.

        No, fiktiotahan kaikki aikamatkailu on, mutta koittakaapa nyt keksiä jotenkin johdonmukainen fiktio!!


      • Anonyymi
        Anonyymi kirjoitti:

        "hyppäämällä menneisyyteen, jolloin itse pysyy samanikäisenä."

        Niin kuin selitin aidosta aikamatkailusta, tässä hypyssä sekoittuu kaksi aikatasoa toisiinsa. Mitä aikamatkustusta sellainen on?

        Siinä nykyhetken ihminen sellaisenaan siinä nykyhetkessä siirtyy aika mukanaan menneisyyteen, jossa on toinen aikataso. Miten nämä aikatasot ovat vuorovaikutuksessa toisiinsa? Kysymyksiä herää valtava määrä.

        No, fiktiotahan kaikki aikamatkailu on, mutta koittakaapa nyt keksiä jotenkin johdonmukainen fiktio!!

        Menneisyyden ihmiset joutuu kokemaan ajan uudelleen siitä hetkestä lähtien, mihin aikamatkaaja saapuu.


      • Anonyymi
        Anonyymi kirjoitti:

        Menneisyyden ihmiset joutuu kokemaan ajan uudelleen siitä hetkestä lähtien, mihin aikamatkaaja saapuu.

        Siinä tapauksessa menneisyyden ihmiset eivät ole menneisyydessä, jos he kokevat "uudelleen" mennyttä aikaa, vaan josskin toisessa ajassa. Olisiko se jokin rinnakkainen aikatodellisuus, jossa tällainen aikamatkailu tapahtuisi?


      • Anonyymi
        Anonyymi kirjoitti:

        Siinä tapauksessa menneisyyden ihmiset eivät ole menneisyydessä, jos he kokevat "uudelleen" mennyttä aikaa, vaan josskin toisessa ajassa. Olisiko se jokin rinnakkainen aikatodellisuus, jossa tällainen aikamatkailu tapahtuisi?

        On vain yksi universumi ,aikalinja, missä aikamatkustus tapahtuu. Sitä vain kirjoitetaan aina uudestaan.


      • Anonyymi
        Anonyymi kirjoitti:

        Paradoksit? Ensin tapahtui jotain ja "sitten" aikamatkustuksen takia sitä ei tapahtunutkaan.

        ??

        Onko olioita, jotka hallitsee kaksi aikaulottuvuutta tai elävät aika-avaruuden ulkopuolella?


      • Anonyymi
        Anonyymi kirjoitti:

        Paradoksit? Ensin tapahtui jotain ja "sitten" aikamatkustuksen takia sitä ei tapahtunutkaan.

        ??

        Vain osa menneisyyden muutoksista ulottaa vaikutuksensa nykyhetkeen.

        Mandela efekti?


      • Anonyymi
        Anonyymi kirjoitti:

        Siinä tapauksessa menneisyyden ihmiset eivät ole menneisyydessä, jos he kokevat "uudelleen" mennyttä aikaa, vaan josskin toisessa ajassa. Olisiko se jokin rinnakkainen aikatodellisuus, jossa tällainen aikamatkailu tapahtuisi?

        "Olisiko se jokin rinnakkainen aikatodellisuus, jossa tällainen aikamatkailu tapahtuisi?"

        Esim. Wheeler-DeWitt yhtälön mukaan mitään rinnakkaisia todellisuuksia ei ole olemassa. Universumin ulkopuolinen tarkkailija näkee vain yhden, ajattoman, staattisen universumin joka ei haaraudu tai koostu rinnakkaisista versioista.

        Jos siis matkustaa menneisyyteen, siellä kaikki aivosolut ja hiukkaset ovat kuten ne menneisyydessä olivat. Tietoisuuden tunneloitumista ajassa taaksepäin ei pysty mistään huomaamaan, koska aivoista ei löydy tulevia tapahtumia koskevia muistikuvia.


      • Anonyymi
        Anonyymi kirjoitti:

        "Olisiko se jokin rinnakkainen aikatodellisuus, jossa tällainen aikamatkailu tapahtuisi?"

        Esim. Wheeler-DeWitt yhtälön mukaan mitään rinnakkaisia todellisuuksia ei ole olemassa. Universumin ulkopuolinen tarkkailija näkee vain yhden, ajattoman, staattisen universumin joka ei haaraudu tai koostu rinnakkaisista versioista.

        Jos siis matkustaa menneisyyteen, siellä kaikki aivosolut ja hiukkaset ovat kuten ne menneisyydessä olivat. Tietoisuuden tunneloitumista ajassa taaksepäin ei pysty mistään huomaamaan, koska aivoista ei löydy tulevia tapahtumia koskevia muistikuvia.

        Universumin ulkopuolinen tarkkailija? Vau

        Älä ota enempää!


      • Anonyymi
        Anonyymi kirjoitti:

        Universumin ulkopuolinen tarkkailija? Vau

        Älä ota enempää!

        Universumin ulkopuolinen tarkkailija...

        Kuluisiko hänenkin aika ?

        Jos hän ensin tarkkailisi 60 - lukua ja sitten 90-lukua. Niin selvästi hänelläkin olisi menneitä ja nyt tapahtuvia tarkkailemisia. Joten hänelläkin kuluisi oma aikansa riippuen mihin hän suuntaa tarkkailemisensa.


      • Anonyymi
        Anonyymi kirjoitti:

        Universumin ulkopuolinen tarkkailija...

        Kuluisiko hänenkin aika ?

        Jos hän ensin tarkkailisi 60 - lukua ja sitten 90-lukua. Niin selvästi hänelläkin olisi menneitä ja nyt tapahtuvia tarkkailemisia. Joten hänelläkin kuluisi oma aikansa riippuen mihin hän suuntaa tarkkailemisensa.

        Universumin ulkopuolinen tarkkailija...

        Ehkä hän tarkkailee kuinka aikamatkustajat muuttavat universumia.


        ...
        Entäs tulevaisuus?

        Onko se lukkoonlyöty?

        Entä vapaa tahtomme tässä tapauksessa?


      • Anonyymi
        Anonyymi kirjoitti:

        Oivallinen päätelmä. Jos olisi kysymys oikeasta aikamatkustuksesta, niin omaan syntymähetkeen matkustaminen tarkoittaisi, että et osaa edes puhua. Kukaan ei syntymähetkellä nimittäin sitä osaa.

        Aikamatkukstuksen fiktiivisissä tarinoissa mennyt aika itse asiassa sekoitetaan aikamatkustajan lähtöajankohdan aikaan. Tämä ei aidossa aikamatkustuksessa pitäisi olla mahdollista. Ei siis voi olla matkaajaa omassa ajassaan niin, että hän otti lähtöhetken ajan mukaansa ja liikkuu sitten menneessä ajassa.

        " aikamatkustus (että siis elelisin syntymäpäiväni uudelleen biologisesti tms.) "

        Uudelleenkokemisen puuttuminen tarkoitti sen poissulkemista. Mistä ei voi puhua, siitä on vaiettava.

        Kiinnostava aikamatkustus on siirtymistä maailman prosessin aikaisempaan tilaan lisäämättä siihen mitään tulevaisuudesta, kuten esim. muistoja (aivojen hermoverkoston kytkentätilaa) sieltä.


      • Anonyymi
        Anonyymi kirjoitti:

        Universumin ulkopuolinen tarkkailija...

        Kuluisiko hänenkin aika ?

        Jos hän ensin tarkkailisi 60 - lukua ja sitten 90-lukua. Niin selvästi hänelläkin olisi menneitä ja nyt tapahtuvia tarkkailemisia. Joten hänelläkin kuluisi oma aikansa riippuen mihin hän suuntaa tarkkailemisensa.

        Universumin ulkopuolisella tarkkailijalla aika ei kulu mihinkään suuntaan, mutta tietoinen kokemus perustuu johonkin muuhun kuin aikaan ja on hyvin erilainen kuin kvanttimekaanisen systeemin sisällä olevilla ihmisillä.

        wiki (artikkelista lomittuminen): "Pagen ja Woottersin päätelmien mukaan aika on emergentti ilmiö, joka johtuu lomittumisen luonteesta ja esiintyy vain universumin sisällä oleville tarkkailijoille. Jokainen jumalankaltainen tarkkailija universumin ulkopuolella näkee puolestaan staattisen, muuttumattoman maailmankaikkeuden juuri kuten Wheeler–DeWitt-yhtälö ennustaa.

        Vuonna 2013 ryhmä tutkijoita Torinosta, Italiasta, teki ensimmäisen testin Pagen ja Woottersin idealle vahvistaen sen, että aika on todellakin emergentti ilmiö "sisäisille" tarkkailijoille, mutta poissaoleva maailmankaikkeuden ulkoisille tarkkailijoille[3][4][5]."


      • Anonyymi
        Anonyymi kirjoitti:

        On vain yksi universumi ,aikalinja, missä aikamatkustus tapahtuu. Sitä vain kirjoitetaan aina uudestaan.

        Paradoksit? Ensin tapahtui jotain ja "sitten" aikamatkustuksen takia sitä ei tapahtunutkaan.

        ??


    • Anonyymi

      Koska aikaa ei ole, ei ole myöskään mahdollista matkata ajassa. Ainoa mahdollisuus siirtyä aiemmin koettuun hetkeen on järjestää joka ikinen atomi, molekyyli, taivaankappale, energiat, fotonit, galaksit jne. täsmälleen siihen asentoon/sijaintiin, jossa ne olivat siinä tietyssä jo tapahtuneessa hetkessä. Silloin pätee se, ettei muistot ole samat kuin lähtöhetkellä, vaan täsmälleen samat kuin menneellä tapahtumahetkellä. Ja koska muita ulottuvuuksia tms. ei ole, jokainen tässä todellisuudessa siirtyy mukana. Perinteisessä aikamatkustuksessa kulunut aika tapahtumineen täytyy tallentua johonkin/jotenkin ja johonkin kyseisen tallenteen hetkeen yksittäinen ihminen voi siirtyä tästä hetkestä. Jos palataan takaisin maanpinnalle, niin ainoa tapa siirtyä menneeseen on muistella jotain jo tapahtunutta.

      • Anonyymi

        Päinvastoin, koska aika on olemassa, jo nykyisin tiedetään, miten aikamatkustus on mahdollista. Tosin vain tulevaisuuteen, kiihdyttämällä matkailija relativistisiin nopeuksiin, jolloin aikadilataatio vie hänet tulevaisuuteen.


      • Anonyymi

        " Ainoa mahdollisuus siirtyä aiemmin koettuun hetkeen on järjestää joka ikinen atomi, molekyyli, taivaankappale, energiat, fotonit, galaksit jne. täsmälleen siihen asentoon/sijaintiin, jossa ne olivat siinä tietyssä jo tapahtuneessa hetkessä. "

        Tässä tapauksessa ei matkusteta ajassa taaksepäin, vaan aika edelleenkin kuluu normaalisti eteenpäin mutta seuraava ajanhetki vain muistuttaa paljon jotain menneisyydessä ollutta ajanhetkeä. Siitä eteenpäin universumi kuitenkaan ei kehity samalla tavalla kuin edellisellä kerralla.

        Aito aikamatkustus tapahtuu siten, että tietoisuus (johon voidaan soveltaa aikaoperaattoria joka antaa aaltomaisen aika-todennäköisyysjakauman) tunneloituu ajassa taaksepäin vaikka muutamia vuosia. Sen jälkeen tietoisuus kokee uudelleen jo kertaalleen koetut asiat, mutta ei voi huomata sitä mistään.


      • Anonyymi
        Anonyymi kirjoitti:

        Päinvastoin, koska aika on olemassa, jo nykyisin tiedetään, miten aikamatkustus on mahdollista. Tosin vain tulevaisuuteen, kiihdyttämällä matkailija relativistisiin nopeuksiin, jolloin aikadilataatio vie hänet tulevaisuuteen.

        Ajan olemassaoloa ei yksikään tiedemies ole todistanut. Aikamatkustusidea on syntynyt kuvitteellisen aikakäsitteen sisällä, tai harhaanjohtamana. Koska sekä aika että aikamatkustus on kuvitteellisia asioita, molemmat voidaan yhteensovittaa keskenään, jonka tuloksena on ajan "todellisuuden" harhaanjohtamana selkeältä tuntuva mahdollisuus matkustaa ajassa.

        Aikadilaatio on selitetty sillä perusteella, että universumissa on ikään kuin ns. vakionopeudella kulkeva aika ja jos havainto on poikkeava, on koetaan tapahtuneen vääristymä ajassa. Ja tämä on virheellinen olettamus. Ihmisen havainnoinnissa fotoneilla on merkittävä rooli, joka on maanpäällisissä olosuhteissa vakiintunut ja muodostanut ihmiselle tapahtumien hahmottamisen "vakionopeuden", joka nähdään myös normaalin ajankulun puitteissa, vaikka kuitenkin kyseessä on kaksi eri asiaa. Kun poikkeavat olosuhteet (valonnopeus, musta aukko...) vaikuttavat fotonien toimintaan, poikkeaa myös fotonien antama informaatio, jonka myötä muuttuu myös aistivarainen, "vakionopeuteen" tottunut kokemus, jonka seurauksena nähdään "aikavääristymä". Tietenkään kuvitteellinen aika ei vääristy mihinkään, vaan aivojen informaation prosessoinnin poikkeama aiheuttaa aistivaraisen vääristymän, joka se tulkitaan virheellisesti "aikavääristymäksi".


      • Anonyymi
        Anonyymi kirjoitti:

        " Ainoa mahdollisuus siirtyä aiemmin koettuun hetkeen on järjestää joka ikinen atomi, molekyyli, taivaankappale, energiat, fotonit, galaksit jne. täsmälleen siihen asentoon/sijaintiin, jossa ne olivat siinä tietyssä jo tapahtuneessa hetkessä. "

        Tässä tapauksessa ei matkusteta ajassa taaksepäin, vaan aika edelleenkin kuluu normaalisti eteenpäin mutta seuraava ajanhetki vain muistuttaa paljon jotain menneisyydessä ollutta ajanhetkeä. Siitä eteenpäin universumi kuitenkaan ei kehity samalla tavalla kuin edellisellä kerralla.

        Aito aikamatkustus tapahtuu siten, että tietoisuus (johon voidaan soveltaa aikaoperaattoria joka antaa aaltomaisen aika-todennäköisyysjakauman) tunneloituu ajassa taaksepäin vaikka muutamia vuosia. Sen jälkeen tietoisuus kokee uudelleen jo kertaalleen koetut asiat, mutta ei voi huomata sitä mistään.

        Mikä on aikaoperaattori? Miten se toimii? Mikä on aaltomainen aika-todennäköisyysjakauma? Miten se toimii, otetaan haltuun ja annetaan? Miten yksi ihminen voidaan siirtää aikaisempaan tapahtumahetkeen? Mikä tekee siitä aikamatkustuksen?


      • Anonyymi
        Anonyymi kirjoitti:

        Mikä on aikaoperaattori? Miten se toimii? Mikä on aaltomainen aika-todennäköisyysjakauma? Miten se toimii, otetaan haltuun ja annetaan? Miten yksi ihminen voidaan siirtää aikaisempaan tapahtumahetkeen? Mikä tekee siitä aikamatkustuksen?

        Aikaoperaattorista löytyy tietoa esim. tuolta:
        https://arxiv.org/pdf/1901.10665.pdf

        Aikaoperaattori on kvanttimekaaninen operaattori, samantyyppinen kuin esim. Hamiltonin operaattori joka antaa hiukkasen tila-todennäköisyysjakauman. Paikan lisäksi myös hiukkasen ajanhetken todennäköisyys on jakautunut. Todennäköisyys löytää se nykyhetkestä on hiukan pienempi kuin 1, ja mahdollisuus että se sijaitsee kaukana tulevaisuudessa tai menneisyydessä on suurempi kuin 0. Tietoisuuden sijaintiin aikajanalla pätee myös samat kvanttimekaniikan lainalaisuudet, joten se voi pienellä todennäköisyydellä hypätä kauas menneisyyteen johtuen fundamentaalisista fysiikan lakien ominaisuuksista.


      • Anonyymi
        Anonyymi kirjoitti:

        Aikaoperaattorista löytyy tietoa esim. tuolta:
        https://arxiv.org/pdf/1901.10665.pdf

        Aikaoperaattori on kvanttimekaaninen operaattori, samantyyppinen kuin esim. Hamiltonin operaattori joka antaa hiukkasen tila-todennäköisyysjakauman. Paikan lisäksi myös hiukkasen ajanhetken todennäköisyys on jakautunut. Todennäköisyys löytää se nykyhetkestä on hiukan pienempi kuin 1, ja mahdollisuus että se sijaitsee kaukana tulevaisuudessa tai menneisyydessä on suurempi kuin 0. Tietoisuuden sijaintiin aikajanalla pätee myös samat kvanttimekaniikan lainalaisuudet, joten se voi pienellä todennäköisyydellä hypätä kauas menneisyyteen johtuen fundamentaalisista fysiikan lakien ominaisuuksista.

        Kerrotko vielä, mikä on se konkreettinen menetelmä, jolla matkaaja lähetään ajassa taaksepäin ja mitä tässä menetelmässä tapahtuu, miten se saadaan aikaan. Hamiltonin operaattori itse on numeroita, kirjaimia ja erikoismerkkejä, sekä liuta teoreettisia selityksiä.


      • Anonyymi
        Anonyymi kirjoitti:

        Ajan olemassaoloa ei yksikään tiedemies ole todistanut. Aikamatkustusidea on syntynyt kuvitteellisen aikakäsitteen sisällä, tai harhaanjohtamana. Koska sekä aika että aikamatkustus on kuvitteellisia asioita, molemmat voidaan yhteensovittaa keskenään, jonka tuloksena on ajan "todellisuuden" harhaanjohtamana selkeältä tuntuva mahdollisuus matkustaa ajassa.

        Aikadilaatio on selitetty sillä perusteella, että universumissa on ikään kuin ns. vakionopeudella kulkeva aika ja jos havainto on poikkeava, on koetaan tapahtuneen vääristymä ajassa. Ja tämä on virheellinen olettamus. Ihmisen havainnoinnissa fotoneilla on merkittävä rooli, joka on maanpäällisissä olosuhteissa vakiintunut ja muodostanut ihmiselle tapahtumien hahmottamisen "vakionopeuden", joka nähdään myös normaalin ajankulun puitteissa, vaikka kuitenkin kyseessä on kaksi eri asiaa. Kun poikkeavat olosuhteet (valonnopeus, musta aukko...) vaikuttavat fotonien toimintaan, poikkeaa myös fotonien antama informaatio, jonka myötä muuttuu myös aistivarainen, "vakionopeuteen" tottunut kokemus, jonka seurauksena nähdään "aikavääristymä". Tietenkään kuvitteellinen aika ei vääristy mihinkään, vaan aivojen informaation prosessoinnin poikkeama aiheuttaa aistivaraisen vääristymän, joka se tulkitaan virheellisesti "aikavääristymäksi".

        "Ajan olemassaoloa ei yksikään tiedemies ole todistanut."

        Pitää paikkaansa, että aikaa ja tilaa ei ole olemassa.
        Hiukkaset sijaitsevat Hilbertin avaruudessa, jonka alkiot eivät ole mitään tila-, tai aika-avaruuden pisteitä. Tila ja aika ovat vain operaattoreiden tuottamia projektioita jotka ihmismieli hahmottaa konkreettisina asioina.


    • Anonyymi

      Montako gaziljardia wekatavua reilussa 13 miljardissa vuodessa on ehtinyt tallentua?

    • Anonyymi

      Tuo ei ole mikään teoria, ei varsinaisesti edes hypoteesi. Ainoastaan väite.

      • Anonyymi

        Miksei se olisi hypoteesi? Samanlaista tieteellistä selitystä eli hypoteesia tarjotaan monelle muullekin asialle, kuten telepatialle tai enkeleille. Ne ovat ainoastaan näennäisesti olemassa, kuten aikakin.


    • Anonyymi

      Luonnossa ei ole aikaa, vaan eteneviä tilanmuutoksia ja vuorovaikutuksia. Säännnöllisisistä jaksolllisista ilmiöistä kuten sydämen lyönneistä ja taivaankappaleiden näennäisten liikkeiden jaksollisudesta teknis-kulturaalisia taipumuksia omannut ihminen on abstrahoinut ajan käsitteen ja edelleen nopeus-käsitteen muutoksen ja duraation tai intervallin osamääränä ja rakentanut tästä edelleen koko klassisen fysiikan differentiaali- ja integraalilaskentoineen. Tämän seurauksena ajasta - luonnon kuvaamisen työkalusta - on tullut itseoikeutettu tekijä lähes kaikkien ilmiöiden tieteellisissä kuvauksissa.

      Voidaankin sanoa, että metodinen harha on johtanut ajan illuusion pitämiseen luonnon yhtenä perustekijänä.

      • Anonyymi

        Voisihan tuohon sanoa että tilamuutokset ja vuorovaikutukset ovat
        ovat sitä aikaa. Nehän ovat tapahtumia ja tapahtumissa on se aika.
        Taivaankappaleiden liike on todellista eikä vahvassa merkityksessä
        näennäistä, kyllä ne liikkuvat siellä välittämättä mitään ihmisten ajattelusta.

        "ihminen on abstrahoinut ajan käsitteen".

        Abstrahointi on tärkeä käsitteenmuodostukseen liittyvä asia, ilman sitä
        rämpisimme sotkuisissa detaljikuvaksissa kuormittumalla toimintakyvyttömiksi.
        Kun aikakäsite on abstrahoitu ja siten abstraktimpi kuin tietty tapahtumakuvaus,
        ei se muuta aikaa epätodelliseksi, abstraktit käsitteet kuvaavat kuitenkin
        todellisuutta. Todellisuudessa on kokonaisuuksia joiden kuvauksiin
        tarvitaan abstraktisia käsitteitä.

        Ajan kieltäjille on joskus kokeiltu kuolema-argumenttia, olet syntynyt joskus,
        elänyt ja kasvanut aikasi, joskus sitten koittaa aika jolloin kuolet.
        Eikä siinä ole mitään illuusiota, raaka tosiasia vain.

        Elämäsi on ollut ajallinen tapahtumaketju ja kehosi on kuin kello jolla mittaat
        aikaasi.

        Joku on sanonut, "ihmisen tarvitsee seurata vain kahta aikaa, toinen on UTC
        aika arkipäiväisiin tapahtumiin ja toinen on kehosi näyttämä aika, josta
        näet miten olet elänyt."

        Et ehkä ole vielä nähnyt metsää puilta.
        Tai tarkemmin, et ole vielä nähnyt taloa kasasta rakennustarvikkeita.

        R.


      • Anonyymi
        Anonyymi kirjoitti:

        Voisihan tuohon sanoa että tilamuutokset ja vuorovaikutukset ovat
        ovat sitä aikaa. Nehän ovat tapahtumia ja tapahtumissa on se aika.
        Taivaankappaleiden liike on todellista eikä vahvassa merkityksessä
        näennäistä, kyllä ne liikkuvat siellä välittämättä mitään ihmisten ajattelusta.

        "ihminen on abstrahoinut ajan käsitteen".

        Abstrahointi on tärkeä käsitteenmuodostukseen liittyvä asia, ilman sitä
        rämpisimme sotkuisissa detaljikuvaksissa kuormittumalla toimintakyvyttömiksi.
        Kun aikakäsite on abstrahoitu ja siten abstraktimpi kuin tietty tapahtumakuvaus,
        ei se muuta aikaa epätodelliseksi, abstraktit käsitteet kuvaavat kuitenkin
        todellisuutta. Todellisuudessa on kokonaisuuksia joiden kuvauksiin
        tarvitaan abstraktisia käsitteitä.

        Ajan kieltäjille on joskus kokeiltu kuolema-argumenttia, olet syntynyt joskus,
        elänyt ja kasvanut aikasi, joskus sitten koittaa aika jolloin kuolet.
        Eikä siinä ole mitään illuusiota, raaka tosiasia vain.

        Elämäsi on ollut ajallinen tapahtumaketju ja kehosi on kuin kello jolla mittaat
        aikaasi.

        Joku on sanonut, "ihmisen tarvitsee seurata vain kahta aikaa, toinen on UTC
        aika arkipäiväisiin tapahtumiin ja toinen on kehosi näyttämä aika, josta
        näet miten olet elänyt."

        Et ehkä ole vielä nähnyt metsää puilta.
        Tai tarkemmin, et ole vielä nähnyt taloa kasasta rakennustarvikkeita.

        R.

        Tapahtumissa itsessään ei ole aikaa, eikä tapahtumat tapahdu ajassa, vaan tapahtumia vain tapahtuu fysiikan lakien ja muiden vaikuttimien perusteella omalla tavallaan ja nopeudellaan. Näitä tapahtumia aistitaan ja tiedostetaan, ja koska ihmisellä on lisäksi kehittynyt muisti, hän tiedostaa yhtäaikaa jo tapahtuneen ja parasta aikaa tapahtuvan, sekä kykenee näiden perusteella ennustamaan. Eli mennyt, nyt tapahtuva ja tuleva kokonaisuutena = keston hahmottamisen rakentuminen. Tämä ei ole sama kuin "aika", mutta ajan keksimiseen johdattanut perusta. "Aika" on uusi, ulkopuolinen, ylimääräinen, keksitty ja kehitetty käsite ja sovellus, joka on vain aivojen kyseisen biologisen prosessin ainoa olemassaoleva verbaalinen vastine. Tämän lisäksi se on myös keino jaksottaa tapahtumia, sekä mitata niiden kestoa, ja tietenkin voimme myös asettaa tapahtumahetkiä tiettyihin kohtiin aikajanalle.

        Aika on siitä mielenkiintoinen keksintö, että se on kyennyt muokkaamaan ajatteluamme ja kokemuksiamme. Ajan avulla nimittäin olemme oppineet kokemaan esim. ajallisia etäisyyksiä. Kun ajattelet vaikka "tunnin päästä", "viime vuonna", tai "10000 vuotta sitten", tai "viikon päästä", niin välittömästi koet ajallisen hetken etäisyyden, eli "milloin" tapahtumahetki on/oli. Aika siis tarjoaa meille uusia verbaalisesti määriteltävissä olevia kokemuksia, kuten "milloin", "kuinka kauan" jne.


      • Anonyymi
        Anonyymi kirjoitti:

        Tapahtumissa itsessään ei ole aikaa, eikä tapahtumat tapahdu ajassa, vaan tapahtumia vain tapahtuu fysiikan lakien ja muiden vaikuttimien perusteella omalla tavallaan ja nopeudellaan. Näitä tapahtumia aistitaan ja tiedostetaan, ja koska ihmisellä on lisäksi kehittynyt muisti, hän tiedostaa yhtäaikaa jo tapahtuneen ja parasta aikaa tapahtuvan, sekä kykenee näiden perusteella ennustamaan. Eli mennyt, nyt tapahtuva ja tuleva kokonaisuutena = keston hahmottamisen rakentuminen. Tämä ei ole sama kuin "aika", mutta ajan keksimiseen johdattanut perusta. "Aika" on uusi, ulkopuolinen, ylimääräinen, keksitty ja kehitetty käsite ja sovellus, joka on vain aivojen kyseisen biologisen prosessin ainoa olemassaoleva verbaalinen vastine. Tämän lisäksi se on myös keino jaksottaa tapahtumia, sekä mitata niiden kestoa, ja tietenkin voimme myös asettaa tapahtumahetkiä tiettyihin kohtiin aikajanalle.

        Aika on siitä mielenkiintoinen keksintö, että se on kyennyt muokkaamaan ajatteluamme ja kokemuksiamme. Ajan avulla nimittäin olemme oppineet kokemaan esim. ajallisia etäisyyksiä. Kun ajattelet vaikka "tunnin päästä", "viime vuonna", tai "10000 vuotta sitten", tai "viikon päästä", niin välittömästi koet ajallisen hetken etäisyyden, eli "milloin" tapahtumahetki on/oli. Aika siis tarjoaa meille uusia verbaalisesti määriteltävissä olevia kokemuksia, kuten "milloin", "kuinka kauan" jne.

        "Tapahtumissa itsessään ei ole aikaa, eikä tapahtumat tapahdu ajassa, vaan
        tapahtumia vain tapahtuu fysiikan lakien ja muiden vaikuttimien perusteella omalla
        tavallaan ja nopeudellaan."

        Tapahtumia siellä ja täällä, tapahtumissa itsessään ei ole aikaa, eikä ne
        tapahdu ajassa, tämä ei ole mitenkään selvä ajatus.

        Mutta, miksi et abstrahoi tapahtumien temporaalisista suhteista aikakäsitettä.
        Siinähän se lähtökohta on tyrkyllä. Jos kerran tapahtumat viuhtovat ympärillä.

        Kun fyysikot eivät tiedä mitä aika on perustasolla, meidän on tyydyttävä
        ilmiötason aikaan, se on kuitenkin oleellinen kuten kuolema-argumentti
        osoittaa.

        Viimeisestä vastauksestasi on jätetty pois "illuusio", no hyvää kehitystä,
        sovellus ja keksitty on vielä jäljellä.

        Kaikki käsitteet ovat tavallaan keksittyjä, käsitteenmuodostuksen tuotteita
        todellisuuden hahmotukseen ja tiedostavaan hallintaan. "Sovellus" on
        kieltämättä huono termi kun tiedetään käsitteiden rooli ja ominaisuudet.

        Meillä ihmisillä on aikakäsite joka tuntuu viittaavan johonkin todellisuudessa
        esiintyvään fyysiseen ominaisuuteen, vastauksestasi on vaikea huomata eroa,
        et erottele aikakäsittettä ja mahdollista fyysistä aikaa selvästi.

        R.


      • Anonyymi
        Anonyymi kirjoitti:

        "Tapahtumissa itsessään ei ole aikaa, eikä tapahtumat tapahdu ajassa, vaan
        tapahtumia vain tapahtuu fysiikan lakien ja muiden vaikuttimien perusteella omalla
        tavallaan ja nopeudellaan."

        Tapahtumia siellä ja täällä, tapahtumissa itsessään ei ole aikaa, eikä ne
        tapahdu ajassa, tämä ei ole mitenkään selvä ajatus.

        Mutta, miksi et abstrahoi tapahtumien temporaalisista suhteista aikakäsitettä.
        Siinähän se lähtökohta on tyrkyllä. Jos kerran tapahtumat viuhtovat ympärillä.

        Kun fyysikot eivät tiedä mitä aika on perustasolla, meidän on tyydyttävä
        ilmiötason aikaan, se on kuitenkin oleellinen kuten kuolema-argumentti
        osoittaa.

        Viimeisestä vastauksestasi on jätetty pois "illuusio", no hyvää kehitystä,
        sovellus ja keksitty on vielä jäljellä.

        Kaikki käsitteet ovat tavallaan keksittyjä, käsitteenmuodostuksen tuotteita
        todellisuuden hahmotukseen ja tiedostavaan hallintaan. "Sovellus" on
        kieltämättä huono termi kun tiedetään käsitteiden rooli ja ominaisuudet.

        Meillä ihmisillä on aikakäsite joka tuntuu viittaavan johonkin todellisuudessa
        esiintyvään fyysiseen ominaisuuteen, vastauksestasi on vaikea huomata eroa,
        et erottele aikakäsittettä ja mahdollista fyysistä aikaa selvästi.

        R.

        "Meillä ihmisillä on aikakäsite joka tuntuu viittaavan johonkin todellisuudessa
        esiintyvään fyysiseen ominaisuuteen"

        "Mutta, miksi et abstrahoi tapahtumien temporaalisista suhteista aikakäsitettä."

        "Voisihan tuohon sanoa että tilamuutokset ja vuorovaikutukset ovat
        ovat sitä aikaa. Nehän ovat tapahtumia ja tapahtumissa on se aika."

        Etsit jatkuvasti ajan todellisuutta sen itsensä sisältä, aikakäsitteen ja -määritelmän sanelemissa rajoissa. Vaikka ajan keksimisen perusta ja aikakäsite ovat 100% sisäsyntyisiä asioita, projisoit sitä väkisinkin itsesi ulkopuolelle (koska tiede sanoo näin). Tämä johtuu siitä, että 'aika' on voimakas psykologinen vaikutin, joka on muokannut ajatteluamme ja kokemuksiamme. Se on niin voimakas, että meritoituneimmat tiedemiehetkin etsii aikaa tuolta ulkoa. Se on saanut mielen uskomaan, että 'aika' on konkreettinen asia ihmisen ulkopuolella. Jotkut uskoo myös aaveisiin, vaikka sekin on 100% sisäsyntyinen ilmiö. Ne vain projisoidaan itsensä ulkopuolelle, "todellisiksi" entiteeteiksi, eikä millään haluta tai kyetä luopua uskomuksesta.

        Haasteenasi on sallia itsellesi lupa hahmottaa erot aivojen biologian rakentamien perusasioiden ja käsitteiden, sekä käsiteperusteisen ajattelun välillä. Kun etsit 'aikaa' oikeasta paikasta, eli aivoista, vasta sitten opit ymmärtämään sen, mitä aika on ja mitä se ei ole, ja missä se on ja missä ei ole. Tapahtumien välillä ei ole aikaa, mutta aikaa soveltamalla voidaan tapahtumia tarkastella ajallisesti. Ihmismielen ulkopuolella tapahtumien välillä on vain ja ainoastaan 'kesto', eli tapahtuma voi olla 'nopea' tai 'hidas'. Esim. hiukkas-antihiukkasparin annihilaatio vs. galaksin muodostuminen. Vain ja ainoastaan ajan keksimisen jälkeen ihminen on voinut soveltaa tapahtumiin 'aikaa' tai kokea tapahtumat 'ajallisesti'. Ethän myöskään voi kokea 'kipua', ellet tunne termiä 'kipu' ja sen merkityssisältöä. Huomio käsitteiden kyvylle muokata ajattelua ja kokemuksia.


      • Anonyymi
        Anonyymi kirjoitti:

        "Meillä ihmisillä on aikakäsite joka tuntuu viittaavan johonkin todellisuudessa
        esiintyvään fyysiseen ominaisuuteen"

        "Mutta, miksi et abstrahoi tapahtumien temporaalisista suhteista aikakäsitettä."

        "Voisihan tuohon sanoa että tilamuutokset ja vuorovaikutukset ovat
        ovat sitä aikaa. Nehän ovat tapahtumia ja tapahtumissa on se aika."

        Etsit jatkuvasti ajan todellisuutta sen itsensä sisältä, aikakäsitteen ja -määritelmän sanelemissa rajoissa. Vaikka ajan keksimisen perusta ja aikakäsite ovat 100% sisäsyntyisiä asioita, projisoit sitä väkisinkin itsesi ulkopuolelle (koska tiede sanoo näin). Tämä johtuu siitä, että 'aika' on voimakas psykologinen vaikutin, joka on muokannut ajatteluamme ja kokemuksiamme. Se on niin voimakas, että meritoituneimmat tiedemiehetkin etsii aikaa tuolta ulkoa. Se on saanut mielen uskomaan, että 'aika' on konkreettinen asia ihmisen ulkopuolella. Jotkut uskoo myös aaveisiin, vaikka sekin on 100% sisäsyntyinen ilmiö. Ne vain projisoidaan itsensä ulkopuolelle, "todellisiksi" entiteeteiksi, eikä millään haluta tai kyetä luopua uskomuksesta.

        Haasteenasi on sallia itsellesi lupa hahmottaa erot aivojen biologian rakentamien perusasioiden ja käsitteiden, sekä käsiteperusteisen ajattelun välillä. Kun etsit 'aikaa' oikeasta paikasta, eli aivoista, vasta sitten opit ymmärtämään sen, mitä aika on ja mitä se ei ole, ja missä se on ja missä ei ole. Tapahtumien välillä ei ole aikaa, mutta aikaa soveltamalla voidaan tapahtumia tarkastella ajallisesti. Ihmismielen ulkopuolella tapahtumien välillä on vain ja ainoastaan 'kesto', eli tapahtuma voi olla 'nopea' tai 'hidas'. Esim. hiukkas-antihiukkasparin annihilaatio vs. galaksin muodostuminen. Vain ja ainoastaan ajan keksimisen jälkeen ihminen on voinut soveltaa tapahtumiin 'aikaa' tai kokea tapahtumat 'ajallisesti'. Ethän myöskään voi kokea 'kipua', ellet tunne termiä 'kipu' ja sen merkityssisältöä. Huomio käsitteiden kyvylle muokata ajattelua ja kokemuksia.

        " Vaikka ajan keksimisen perusta ja aikakäsite ovat 100% sisäsyntyisiä asioita,
        projisoit sitä väkisinkin itsesi ulkopuolelle (koska tiede sanoo näin)."

        Käsitteenmuodostus on tietenkin aivojen tuote, täysin automaattinen
        tapahtuma joka vaikuttaa koko elämän ajan.
        Katson kuitenkin että käsitteet viittaavat myös ulkotodellisuuteen eikä
        vain aaveharhakuviin, aaveita varten meillä on "harhan" käsite.

        Käsitteillä kuitenkin ihmiset ajattelevat ja hahmottavat itseään ja todellisuutta.
        Hyväksyn että on jatkuva vuorovaikutus käsitteiden ja todellisuuden välillä,
        myös käsitteet tulevat tarkemmiksi kokemusten myötä, jotkut käsitteet
        poistuvat käytöstä ja uusia kokeillaan tieteen ja kokemusten kautta.

        Tuo ilmaisu "100% sisäsyntyisiä asioita" on hankala ja epäselvä ilmaisu,
        en katso, että sillä olisi vahvoja implikaatioita siihen mikä aikakäsitteen
        rooli ja asema olisi.

        Olen hyvin tietoinen siitä miten aivot toimivat ja millainen käsitteiden rooli
        siinä on.

        Millainen aika on ihmismielen ulkopuolella tai ehkä ilman ihmistä?

        Koska tällöin ei olisi aikakäsitettä, mutta kuitenkin olisi tapahtumia,
        tapahtumilla olisi jotain ominaisuuksia ja suhteita toisiin tapahtumiin
        nähden.

        Jätetään BigBang pois ja katsotaan että tapahtumat eivät tapahtuneet
        yhdessä kasassa, vaan niillä on avaruus-suhteita toisiin nähden ja eivät
        yhtäaikaa samassa rysäyksessä, vaan niillä oli temporalisia, ajallisia,
        suhteita toisiinsa nähden.

        "Kesto" on yksi ajallisista suhteista, tosin se voisi kuvata tapahtumaketjun
        tilannetta. Annihilaatio voi olla myös tapahtumaketju, ainakin galaksin
        muodostuminen on. Kesto viittaa tapahtuuko tapahtuma1 nopeammin
        kuin tapahtuma2. Tuo "nopeammin" viittaa suhteeseen tapahtumien
        välillä eikä ole tapahtuman ominaisuus.

        On muitakin ajallisia suhteita, ennen-jälkeen, samanaikainen, eriaikainen
        yhtäaikainen yms. Nämä ajalliset suhteet eivät ole itse aika vaan lähinnä
        ajan "primitiiveja".
        Primitiiveja samalla tavoin kuin rakennustarvikkeiden kasa on talon primitiivi.

        Luonnontieteissä kellot ja aikakäsite ovat tärkeitä siksi että ne kertovat
        tapahtumajärjestyksen.

        Miten sitten kivun ja kiputermin suhde on?

        Vauvat kokevat kipua, vaikka heillä ei olisi mitään termiä käytössä,
        kun kiputermi tulee myöhemmin käyttöön, silloin osataan kuvailla sillä
        missä kipu, miltä se tuntuu yms

        Eläimet tuntevat kipua, onko niillä kiputermiä joillakin ehkä, toisilla ei.
        Ainakin kissan kielessä on kipuun viittaava termi jota käytetään
        vierasta eläintä tai ihmistä kohtaan, mutta ei omiin ihmisiin.

        Koirilla on metsästys-termiin liittyvä termi joka viittaa saaliin kipuun ja sen
        tuottamiseen.
        Susilla on omaan loukkaantumiseen liittyvä termi joka vaikuttaa toisten
        susien käyttäytymiseen heti tai myöhemmin.

        "Huomio käsitteiden kyvylle muokata ajattelua ja kokemuksia."

        Huomioitu on ja sen lisäksi kokemusten vaikutus käsitteiden sisältöön.
        Ei tässä ole vain yksisuuntaista muokkausta.

        R.


      • Anonyymi
        Anonyymi kirjoitti:

        " Vaikka ajan keksimisen perusta ja aikakäsite ovat 100% sisäsyntyisiä asioita,
        projisoit sitä väkisinkin itsesi ulkopuolelle (koska tiede sanoo näin)."

        Käsitteenmuodostus on tietenkin aivojen tuote, täysin automaattinen
        tapahtuma joka vaikuttaa koko elämän ajan.
        Katson kuitenkin että käsitteet viittaavat myös ulkotodellisuuteen eikä
        vain aaveharhakuviin, aaveita varten meillä on "harhan" käsite.

        Käsitteillä kuitenkin ihmiset ajattelevat ja hahmottavat itseään ja todellisuutta.
        Hyväksyn että on jatkuva vuorovaikutus käsitteiden ja todellisuuden välillä,
        myös käsitteet tulevat tarkemmiksi kokemusten myötä, jotkut käsitteet
        poistuvat käytöstä ja uusia kokeillaan tieteen ja kokemusten kautta.

        Tuo ilmaisu "100% sisäsyntyisiä asioita" on hankala ja epäselvä ilmaisu,
        en katso, että sillä olisi vahvoja implikaatioita siihen mikä aikakäsitteen
        rooli ja asema olisi.

        Olen hyvin tietoinen siitä miten aivot toimivat ja millainen käsitteiden rooli
        siinä on.

        Millainen aika on ihmismielen ulkopuolella tai ehkä ilman ihmistä?

        Koska tällöin ei olisi aikakäsitettä, mutta kuitenkin olisi tapahtumia,
        tapahtumilla olisi jotain ominaisuuksia ja suhteita toisiin tapahtumiin
        nähden.

        Jätetään BigBang pois ja katsotaan että tapahtumat eivät tapahtuneet
        yhdessä kasassa, vaan niillä on avaruus-suhteita toisiin nähden ja eivät
        yhtäaikaa samassa rysäyksessä, vaan niillä oli temporalisia, ajallisia,
        suhteita toisiinsa nähden.

        "Kesto" on yksi ajallisista suhteista, tosin se voisi kuvata tapahtumaketjun
        tilannetta. Annihilaatio voi olla myös tapahtumaketju, ainakin galaksin
        muodostuminen on. Kesto viittaa tapahtuuko tapahtuma1 nopeammin
        kuin tapahtuma2. Tuo "nopeammin" viittaa suhteeseen tapahtumien
        välillä eikä ole tapahtuman ominaisuus.

        On muitakin ajallisia suhteita, ennen-jälkeen, samanaikainen, eriaikainen
        yhtäaikainen yms. Nämä ajalliset suhteet eivät ole itse aika vaan lähinnä
        ajan "primitiiveja".
        Primitiiveja samalla tavoin kuin rakennustarvikkeiden kasa on talon primitiivi.

        Luonnontieteissä kellot ja aikakäsite ovat tärkeitä siksi että ne kertovat
        tapahtumajärjestyksen.

        Miten sitten kivun ja kiputermin suhde on?

        Vauvat kokevat kipua, vaikka heillä ei olisi mitään termiä käytössä,
        kun kiputermi tulee myöhemmin käyttöön, silloin osataan kuvailla sillä
        missä kipu, miltä se tuntuu yms

        Eläimet tuntevat kipua, onko niillä kiputermiä joillakin ehkä, toisilla ei.
        Ainakin kissan kielessä on kipuun viittaava termi jota käytetään
        vierasta eläintä tai ihmistä kohtaan, mutta ei omiin ihmisiin.

        Koirilla on metsästys-termiin liittyvä termi joka viittaa saaliin kipuun ja sen
        tuottamiseen.
        Susilla on omaan loukkaantumiseen liittyvä termi joka vaikuttaa toisten
        susien käyttäytymiseen heti tai myöhemmin.

        "Huomio käsitteiden kyvylle muokata ajattelua ja kokemuksia."

        Huomioitu on ja sen lisäksi kokemusten vaikutus käsitteiden sisältöön.
        Ei tässä ole vain yksisuuntaista muokkausta.

        R.

        Vielä ajatuksia ajasta.

        Fyysikkojen mukaan todellisuus muodostuu tapahtumien kokonaisuudesta,
        ei niinkään objektien.
        Kaikki kokemukset jotka vähäänkään viittavat aikaan tulevat ilmiötason
        kokemuksista.
        Todellisuuden perustason aikarakenteet, jos niitä on, ovat vielä tieteeltä
        piilossa.

        Ilmiötason tapahtumien kokonaisuus on niin suuri ettei ihminen pysty
        hahmottamaan sitä, jo shakkipelin tapahtumat ovat ihmiselle liikaa, tarvitaan
        tietokoneiden jumalakatsetta analysoimaan sitä ihmisen puolesta.

        Samanlainen ilmiö on kaikkialla, politiikan, talouden, sodan ja tieteen tapahtumat
        ovat meille lähes liikaa.
        On kuitenkin menetelmä ratkaista tämä puute, abstraktisten käsitteistöjen
        käyttämät mallit ja ennusteet.
        Suurten kokonaisuuksien käyttäytymistä ei voida ymmärtää konkreettisella
        käsitteillä vaan ratkaisu löytyy abstraktisista käsitteistä.
        Tämä näkyy luonnontieteiden kehityksessä ja yleensä tiedon lisääntymisessä.

        On totta, ettemme havaitse aikaa tai emme pysty mittaamaan sitä suoraan,
        tämä voi olla epäselvä kommentti, mutta siltä monesta on tuntunut.
        Tämän ei tarvitse olla kohtalokasta ajalle, se voi olla ihmiseltä piilossa
        tapahtumien kokonaisuuden ominaisuutena, niin ettei ihminen pysty sitä
        sieltä tunnistamaan.

        Sanoisin että tapahtumien kokonaisuudessa on ajan piilopaikka koska mitään
        muuta ei ole.
        Se ei löydy kelloista sellaisenaan, eikä jaksoista, ne ovat tietynlaisia vain
        tapahtumaketjuja eivät itse aikaa.

        Toisaalta on helppo sanoa mitkä väitteet ajasta ovat selvästi vääriä,
        aika ei ole vain käsite, sopimus, keksitty, sovellus ja vielä termi.

        Meillä on aikakäsite, se on kuitenkin ajatustiivistymä siitä mitä tunnumme
        tietävän ajasta, ajatus ajasta, sillä voimme tehdä sovelluksia vaikka
        aikapommin. Ei siitä seuraa että aika on sovellus.

        Vanha vertaus: Meillä on lehmäkäsite, ymmärrys lehmästä. Tuo käsite on
        aivoissamme, sitä ei voi lypsää eikä syödä.

        Fyysinen lehmä on se mihin lehmäkäsite viittaa, sitä voi lypsää ja sen
        voi syödä.

        Niille, jotka toistavat vuodesta toiseen ettei aikaa ole, miten stten on
        suhtauduttava kuolema-argumenttiin, yksikään kieltäjä ei ole antanut
        minkäänlaista kommenttia siihen.

        R.


      • Anonyymi
        Anonyymi kirjoitti:

        "Tapahtumissa itsessään ei ole aikaa, eikä tapahtumat tapahdu ajassa, vaan
        tapahtumia vain tapahtuu fysiikan lakien ja muiden vaikuttimien perusteella omalla
        tavallaan ja nopeudellaan."

        Tapahtumia siellä ja täällä, tapahtumissa itsessään ei ole aikaa, eikä ne
        tapahdu ajassa, tämä ei ole mitenkään selvä ajatus.

        Mutta, miksi et abstrahoi tapahtumien temporaalisista suhteista aikakäsitettä.
        Siinähän se lähtökohta on tyrkyllä. Jos kerran tapahtumat viuhtovat ympärillä.

        Kun fyysikot eivät tiedä mitä aika on perustasolla, meidän on tyydyttävä
        ilmiötason aikaan, se on kuitenkin oleellinen kuten kuolema-argumentti
        osoittaa.

        Viimeisestä vastauksestasi on jätetty pois "illuusio", no hyvää kehitystä,
        sovellus ja keksitty on vielä jäljellä.

        Kaikki käsitteet ovat tavallaan keksittyjä, käsitteenmuodostuksen tuotteita
        todellisuuden hahmotukseen ja tiedostavaan hallintaan. "Sovellus" on
        kieltämättä huono termi kun tiedetään käsitteiden rooli ja ominaisuudet.

        Meillä ihmisillä on aikakäsite joka tuntuu viittaavan johonkin todellisuudessa
        esiintyvään fyysiseen ominaisuuteen, vastauksestasi on vaikea huomata eroa,
        et erottele aikakäsittettä ja mahdollista fyysistä aikaa selvästi.

        R.

        "Kun fyysikot eivät tiedä mitä aika on perustasolla, meidän on tyydyttävä
        ilmiötason aikaan, se on kuitenkin oleellinen kuten kuolema-argumentti
        osoittaa."

        Eivät tiedä, kun ei ole mitään tietämistä. Jotkut prosessit vain etenevät hitaammin, jotkut nopeammin, joten ihminen on alkanut suhteuttaa suhteellisia kestoaikoja keksimiinsä kelloihin ja kalentereihin, ja lopulta rakentanut kaikki luonnon selityksensäkin ajan funktioiksi. Tälle "selitämiselle" tullaan vielä hyväntahtoisessti hymyilemään (ensin kirjoitin: makeasti naureskelemaan), kun luonto ymmärretään oikein.


    • Anonyymi

      Minun ei tarvitse väittää väitettäsi vastaan väitettä koska väitteesi on vain väite.

      • Anonyymi

        Ei sitä kysytty, vaan miten voit >todistaa< aloituksen väitteen vääräksi.


      • Anonyymi
        Anonyymi kirjoitti:

        Ei sitä kysytty, vaan miten voit >todistaa< aloituksen väitteen vääräksi.

        Ensin pitää sopia falsifioinnin kriteereistä. Esim. jos havaitaan kellon viisarin liikkuvan eteenpäin, voidaan väitettä pitää falsifioituna. Kellon pysähtyminen toisaalta ei todista väitettä oikeaksi.


      • Anonyymi
        Anonyymi kirjoitti:

        Ensin pitää sopia falsifioinnin kriteereistä. Esim. jos havaitaan kellon viisarin liikkuvan eteenpäin, voidaan väitettä pitää falsifioituna. Kellon pysähtyminen toisaalta ei todista väitettä oikeaksi.

        Kello ei ole aika, eikä todista aikaa. Se on vain mekaaninen tai elektroninen laite, joka toimii sopimuksen mukaista tahtia. Viisarin liike ja pysähtyminen ovat kummatkin yhtä merkityksettömiä.


      • Anonyymi
        Anonyymi kirjoitti:

        Kello ei ole aika, eikä todista aikaa. Se on vain mekaaninen tai elektroninen laite, joka toimii sopimuksen mukaista tahtia. Viisarin liike ja pysähtyminen ovat kummatkin yhtä merkityksettömiä.

        Mutta jos aikaa ei ole olemassa, ei ole olemassa tahtiakaan eikä kellon viisari tällöin liiku. Viisarin liike osoittaa ajan olemassaolon.
        Mitä koejärjestelyä ehdotat, jos kellon viisarin katsominen ei käy?


      • Anonyymi
        Anonyymi kirjoitti:

        Mutta jos aikaa ei ole olemassa, ei ole olemassa tahtiakaan eikä kellon viisari tällöin liiku. Viisarin liike osoittaa ajan olemassaolon.
        Mitä koejärjestelyä ehdotat, jos kellon viisarin katsominen ei käy?

        Olet ihan kujalla XD Aloituksen väite on hyvin yksinkertainen todistaa vääräksI, kun ymmärtää väitteen puutteellisuuden.

        Aika ON OLEMASSA keksintönä, terminä, käsitteenä ja sovelluksena. Aikaa käytetään lähes kaikkialla maailmassa mittaukseen, ajankohtien asettamiseen, tieteissä, kilpaurheilussa jne. Vaikka sinun kellosi olisi pysähtynyt, ajan soveltaminen ja muu käyttö jatkuu edelleen muualla. Aika on siis todistettavasti olemassa.

        Täydennetään aloitusta: "Jos väitän, ettei aikaa ole [universumissa tietynlaisena itsenäisenä vaikuttavana ja toimivana komponenttina tai voimana] olemassa, miten todistat väitteeni vääräksi?"

        Tässä vaiheessa sinun olisi pitänyt jo saada laajempaa kuvaa ajasta, mitä se on ja mitä se ei ole. Mikä on vastauksesi täydennettyyn väitteeseen?


      • Anonyymi
        Anonyymi kirjoitti:

        Olet ihan kujalla XD Aloituksen väite on hyvin yksinkertainen todistaa vääräksI, kun ymmärtää väitteen puutteellisuuden.

        Aika ON OLEMASSA keksintönä, terminä, käsitteenä ja sovelluksena. Aikaa käytetään lähes kaikkialla maailmassa mittaukseen, ajankohtien asettamiseen, tieteissä, kilpaurheilussa jne. Vaikka sinun kellosi olisi pysähtynyt, ajan soveltaminen ja muu käyttö jatkuu edelleen muualla. Aika on siis todistettavasti olemassa.

        Täydennetään aloitusta: "Jos väitän, ettei aikaa ole [universumissa tietynlaisena itsenäisenä vaikuttavana ja toimivana komponenttina tai voimana] olemassa, miten todistat väitteeni vääräksi?"

        Tässä vaiheessa sinun olisi pitänyt jo saada laajempaa kuvaa ajasta, mitä se on ja mitä se ei ole. Mikä on vastauksesi täydennettyyn väitteeseen?

        SI-järjestelmässä aika on määritelty cesium 133 -atomin värähtelytaajuudesta. Atomithan värähtelevät kyllä, vaikka niitä ei kukaan soveltaisi tai mittaisi. Jos aikaa ei olisi olemassa, universumi olisi staattinen ja muuttumaton ja siellä ei mikään värähtelisi.


    • Anonyymi

      Elisapetti kuoli sentään ihan vanhuuteen. Ei vanhuuteensa.

      • Anonyymi

        Jos ihan pilkuntarkkoja ollaan, vanhuuteen ei voi kuolla, koska se on vain käsite ja määritelmä, joka kertoo tietystä vaiheesta ja sen olosuhteista ihmisen elinkaaressa. Ts. käsite ei kykene sellaisenaan vaikuttamaan ihmisen elintoimintoihin. Sen sijaan kuollaan esim. 'vanhuuttaan', eli tiettyjen olosuhteiden vuoksi.


    • Anonyymi

      On syytä oppia erottamaan aika ja keston havaitseminen toisistaan, sillä ne todellakin ovat eri asioita. Aika on mekaanista tai elektronista jatkuvaa etenevän ajan mittaamista, kun taas keston havaitseminen on aivojen biologista toimintaa, joka perustaltaan ei ole aikaa, eikä sidoksissa aikaan.

      Myöskään universumin tapahtumat eivät tunne aikaa, eikä ole sidoksissa siihen. Tapahtumilla on omat fysikaaliset vaihtelevat ominaisuutensa ja rajansa, joiden mukaan universumi toimii. Vain ihminen soveltaa havaitsemiinsa tapahtumiin aikaa.

      Koko "aikamysteeri" muodostuu siitä, että ihmiset ovat sitoutuneet koordinoimaan lähes kaikkea elämässä kellonajan mukaan. Näin tapahtumien keston hahmottaminen ja aika synkronoituvat yhteen ja siksi tapahtumat koetaan ajassa tai ajallisesti.

      Minkäänlaista aikailmentymää tai entiteettiä universumissa ei ole koskaan ollut olemassa, eikä tule olemaan.

      • Anonyymi

        Puhut nyt ajan ja keston havaitsemisen erottamisesta toisistaan.
        Aikaisemmin sanoit että ennen ihmistä oli kestoa.

        Minkä tahansa havaitseminen liittyy aivoihin, siitä ei liene erimielisyyttä.
        Aivojen toimintaan kyllä liittyy aikakin, vaikka osien ajalliseen tahdistamiseen
        toisiinsa nähden. aivot eivät ole ajaton systeemi, miten muuten aivoista
        saataisiin aivosähkökäyrä.

        Kesto ja keston havaitseminen ovat eri asioita.
        Kesto on tapahtumien ajallista pituutta, joten se aika on edelleen
        mukana kestossakin.

        Yleensä tapahtumissa on aina aika mukana, eikä se riipu ihmisen aivoista,
        maapallo on aikoinaan muodostunut tietyistä tapahtumista ilman aivoja.
        Dinosaurusten aikana oli jo kestoa ja muitakin ajallisluonteisia suhteita
        silloisten tapahtumien välillä.

        "Myöskään universumin tapahtumat eivät tunne aikaa, eikä ole sidoksissa
        siihen."

        Hauska ilmaisu, jos aika on tapahtumissa niin mitään tuntemista ei tarvita,
        eikä olla mitenkään sidoksissakaan aikaan.

        Suhteuttamisen ja suhteen välillä on ero, ne eivät ole sama asia,
        kellon kanssa mitatessa jonkin tapahtuman kestoa, se suhteutetaan
        kellon tapahtumien kanssa, saadaan sitten tulos vaikka tunteina.

        Suhteet eivät tarvitse suhteuttajaa, jo se että joku dinosaurus oli
        nopeampi kuin toinen, tarkoittaa vain että ero oli dinojen ominaisuuksissa,
        ja tietysti sitä että aika oli tässä mukana, tavalla josta olemme eri mieltä.

        Tapahtumissa on aina ajalliset suhteet mukana.
        On sanottu että kun ei ole tapahtumia, ei enää ole aikaa.

        Tai ehkä kun ei ole tapahtumia, ei enää ole tapahtumien välisiä ajallisia
        suhteitakaan.


        Miten kellot ja yhteiskunta sitten.
        Kellot ja aikajärjestelmä tahdistavat yhteiskunnan tapahtumat.

        Vuosia sitten eräässä tv-dokumentissa kerrotttiin minkälaista oli
        junamatkustaminen ennen aikavyöhykkeiden keksimistä.
        Aikataulut olivat olemassa mutta ne kertoivat koska saavutaan kaupunkiin
        sen aurinkoajassa, saapuminen seuraavaan kaupunkiin oli sen paikallisen
        aurinkoajan aikaan.

        Matkustaja olisi pahimmillaan tarvinnut kellon jokaiselle kaupungille erikseen
        jotta tietäisi saapumisajat ja jatkoyhteyksien lähtöajat.

        Kun aikavyöhykkeessä kaupunkien kellot kävivät samaa aikaa,
        aikataulu ongelmat hävisivät samantien.

        Kyse on vain yhteiskunnan tapahtumien tahdistamisesta, tiedät milloin
        uutiset tulevat, milloin mennään kouluun/töihin ja että aikataulut toimivat.

        Sitten kun olet eläkkeellä, voit heittää kellot roskikseen ja elää vain
        paikallista aurinkoaikaa.

        R.


      • Anonyymi
        Anonyymi kirjoitti:

        Puhut nyt ajan ja keston havaitsemisen erottamisesta toisistaan.
        Aikaisemmin sanoit että ennen ihmistä oli kestoa.

        Minkä tahansa havaitseminen liittyy aivoihin, siitä ei liene erimielisyyttä.
        Aivojen toimintaan kyllä liittyy aikakin, vaikka osien ajalliseen tahdistamiseen
        toisiinsa nähden. aivot eivät ole ajaton systeemi, miten muuten aivoista
        saataisiin aivosähkökäyrä.

        Kesto ja keston havaitseminen ovat eri asioita.
        Kesto on tapahtumien ajallista pituutta, joten se aika on edelleen
        mukana kestossakin.

        Yleensä tapahtumissa on aina aika mukana, eikä se riipu ihmisen aivoista,
        maapallo on aikoinaan muodostunut tietyistä tapahtumista ilman aivoja.
        Dinosaurusten aikana oli jo kestoa ja muitakin ajallisluonteisia suhteita
        silloisten tapahtumien välillä.

        "Myöskään universumin tapahtumat eivät tunne aikaa, eikä ole sidoksissa
        siihen."

        Hauska ilmaisu, jos aika on tapahtumissa niin mitään tuntemista ei tarvita,
        eikä olla mitenkään sidoksissakaan aikaan.

        Suhteuttamisen ja suhteen välillä on ero, ne eivät ole sama asia,
        kellon kanssa mitatessa jonkin tapahtuman kestoa, se suhteutetaan
        kellon tapahtumien kanssa, saadaan sitten tulos vaikka tunteina.

        Suhteet eivät tarvitse suhteuttajaa, jo se että joku dinosaurus oli
        nopeampi kuin toinen, tarkoittaa vain että ero oli dinojen ominaisuuksissa,
        ja tietysti sitä että aika oli tässä mukana, tavalla josta olemme eri mieltä.

        Tapahtumissa on aina ajalliset suhteet mukana.
        On sanottu että kun ei ole tapahtumia, ei enää ole aikaa.

        Tai ehkä kun ei ole tapahtumia, ei enää ole tapahtumien välisiä ajallisia
        suhteitakaan.


        Miten kellot ja yhteiskunta sitten.
        Kellot ja aikajärjestelmä tahdistavat yhteiskunnan tapahtumat.

        Vuosia sitten eräässä tv-dokumentissa kerrotttiin minkälaista oli
        junamatkustaminen ennen aikavyöhykkeiden keksimistä.
        Aikataulut olivat olemassa mutta ne kertoivat koska saavutaan kaupunkiin
        sen aurinkoajassa, saapuminen seuraavaan kaupunkiin oli sen paikallisen
        aurinkoajan aikaan.

        Matkustaja olisi pahimmillaan tarvinnut kellon jokaiselle kaupungille erikseen
        jotta tietäisi saapumisajat ja jatkoyhteyksien lähtöajat.

        Kun aikavyöhykkeessä kaupunkien kellot kävivät samaa aikaa,
        aikataulu ongelmat hävisivät samantien.

        Kyse on vain yhteiskunnan tapahtumien tahdistamisesta, tiedät milloin
        uutiset tulevat, milloin mennään kouluun/töihin ja että aikataulut toimivat.

        Sitten kun olet eläkkeellä, voit heittää kellot roskikseen ja elää vain
        paikallista aurinkoaikaa.

        R.

        "Puhut nyt ajan ja keston havaitsemisen erottamisesta toisistaan.
        Aikaisemmin sanoit että ennen ihmistä oli kestoa."

        Pieni korjaus tuohon: puhun ajasta käsitteenä ja keston havaitsemisesta aivojen ominaisuutena. Ettei sekoteta näitä asioita yhdeksi.

        Tapahtumilla on kestonsa oli ihmistä tai ei.

        Tapahtumilla itsellään ei ole ajallisia suhteita. Vain ihminen näkee niissä sellaisia soveltamalla niihin aikaa.

        "Hauska ilmaisu, jos aika on tapahtumissa niin mitään tuntemista ei tarvita,
        eikä olla mitenkään sidoksissakaan aikaan."

        Mutta tapahtumissa itsessään ei ole aikaa. Aika on sovellus, jolla tapahtumia mitataan ja määritellään. Aika nähdään tapahtumissa, koska keston hahmottaminen ja aika on synkronoitunut yhdeksi tavaksi hahmottaa tapahtumia.


      • Anonyymi
        Anonyymi kirjoitti:

        "Puhut nyt ajan ja keston havaitsemisen erottamisesta toisistaan.
        Aikaisemmin sanoit että ennen ihmistä oli kestoa."

        Pieni korjaus tuohon: puhun ajasta käsitteenä ja keston havaitsemisesta aivojen ominaisuutena. Ettei sekoteta näitä asioita yhdeksi.

        Tapahtumilla on kestonsa oli ihmistä tai ei.

        Tapahtumilla itsellään ei ole ajallisia suhteita. Vain ihminen näkee niissä sellaisia soveltamalla niihin aikaa.

        "Hauska ilmaisu, jos aika on tapahtumissa niin mitään tuntemista ei tarvita,
        eikä olla mitenkään sidoksissakaan aikaan."

        Mutta tapahtumissa itsessään ei ole aikaa. Aika on sovellus, jolla tapahtumia mitataan ja määritellään. Aika nähdään tapahtumissa, koska keston hahmottaminen ja aika on synkronoitunut yhdeksi tavaksi hahmottaa tapahtumia.

        "Tapahtumilla on kestonsa oli ihmistä tai ei."

        Jätetään keston havaitseminen pois.
        Käytät tässä "kestoa" sinä merkityksessä kuin se suomen kielessä on.

        "synkronoitunut" tarkoittaa vain ajoitettua, ainakin tietotekniikassa.
        Jos se esiintyy yhteiskunnassa vääristävänä tekijänä, se on sitten
        uusi määritelmä.

        Aikakäsite on kuitenkin aika yksinkertainen käsite, sen logiikka on aika
        suoraviivaista ja selvää.
        Se, mikä tekee "aikamysteerin" on käsitteen korkeassa abstraktiotasossa
        ja siinä että sen "mekanismia" etsitään sieltä sun täältä.

        Aikakäsitteen työmääritelmän pystyy kertomaan kuudella sanalla, sen
        aukeaminen on abstraktisuuden takia toinen juttu.

        "Kesto" on yksi käsite millä aikakäsitettä on määritelty, toiset olisivat
        "ajoitus" ja "tapahtumajärjestys".

        Tapahtumia on kaikkialla, kun ihminen kävelee silloin on ajallisia tapahtumia
        meneillään, kun ihminen pysähtyy siinäkin on ajallisia tapahtumia, kun
        ihminen "ei tee mitään" siinäkin on ajallisia tapahtumia.

        Sama juttu kun dinosaurus kävelee siinä on samat asiat kuin ihmisen
        vastaavassa toiminnassa.

        Ihmisen ja dinon tekemiset ovat samanlaisia aikakäsitteen ajallisuuden ja
        avaruuden suhteiden kannalta.

        Kestokäsite on kuitenkin määritelty tapahtumasuhteiden avulla, se on vain
        yksi vastaavista käsitteistä, siinä ei ole mitään erikoista, se implikoi
        loppujen lopuksi vain tapahtumien välisiä ajallisia suhteita.

        Aikakäsitteen yksinkertaisuuden takia voimme jättää pois yhteiskunnan
        vääristyneen aikakäsitteen käytön se, ei näy mitenkään aikakäsitteen
        määrittelyssä.

        "Minkäänlaista aikailmentymää tai entiteettiä universumissa ei ole koskaan ollut

        olemassa, eikä tule olemaan."

        En tiedä mitä tarkoitat aikailmentymällä tai (aika)entiteetillä, enkä osaa sanoa
        noin laajalla aikaperspektiivilla mitä universumissa tulee tapahtumaan.

        Mutta keskustelun suhteen yksi tärkeä asia puuttuu.

        Mikä on sinun aikakäsitteen määritelmäsi?

        Voit antaa jonkinlaisen työmääritelmän tai karkean hahmotelman.

        R.


    • Anonyymi

      aika liittyy läheisesti xaxaan. ajan yläkäsite on hyhy. aika ilmenee ihmiselle ququna.

      nämä oudot käsitteet aukeavat ihmisille parin sadan vuoden kuluttua :-)

      ... ..

      • Anonyymi

        "aika liittyy läheisesti xaxaan. ajan yläkäsite on hyhy. aika ilmenee ihmiselle ququna."

        Kun suomut putoavat silmiltäsi näet ququn täyttävä maan.
        Puhdistunut katseesi näyttää myöhemmin xaxan olevan kiinni ququissa.

        Verhoutunut hyhy vilahtaa joskus kun katson maailmaa xaxa-lasien läpi.

        r.


      • Anonyymi
        Anonyymi kirjoitti:

        "aika liittyy läheisesti xaxaan. ajan yläkäsite on hyhy. aika ilmenee ihmiselle ququna."

        Kun suomut putoavat silmiltäsi näet ququn täyttävä maan.
        Puhdistunut katseesi näyttää myöhemmin xaxan olevan kiinni ququissa.

        Verhoutunut hyhy vilahtaa joskus kun katson maailmaa xaxa-lasien läpi.

        r.

        "aika liittyy läheisesti xaxaan. ajan yläkäsite on hyhy. aika ilmenee ihmiselle ququna. "

        Tuo on todella vaikea, ei ihme että se aukeaa vasta 200 vuoden kuluttua,

        Jos antaisin vähän viilatun ratkaisu yrityksen siitä.

        Ququ on kaikkialla.
        Ququ muodostuu xaxassa.
        Xaxa muodostuu hyhyssä.
        Hyhy on xaxan verhoama.

        r.


    • Anonyymi

      Liike on aikaa.

      • Anonyymi

        Liikettä mitataan ajalla.


    • Anonyymi

      Jos aikaa ei olisi olemassa niin ei olisi mitään, ei sinuakaan.
      Zen-kysymys: mitä on ei mitään? Zen-vastaus; Ei mitään on mitä tahansa, missä mikään ei vaikuta mihinkään. Todiste: kvanttiteoria, joka on kokeellisesti oikeaksi todennettu.

    • Anonyymi

      Vähän myöhässä mutta kiitos kaikista vastauksista!

    • Anonyymi

      Minkälainen on maailma kun aika supistuu pois - ja paikka jää?

      • Anonyymi

        Mikään ei muutu silloin. Ei edes mielentilat. Jos tapahtuisi pienikin muutos niin aika kulkisi jälleen eteenpäin. Eli aika ennen sitä muutosta ja aika sen muutoksen jälkeen.


      • Anonyymi

        Alustan edellistä lisää.

        Minulla on omakohtaista kokemusta maailmasta jossa ei ole aikaa: Unessa.

        En tiedä kuin monet näkevät unia siten, että tekevät niistä havaintoa nähdessään niitä - minä teen. Mm. en kuule ääniä - näen ne. Äänen kuuleminen edellyttää ajan olemassa oloa. Unikin koostuu erillisistä peräkkäisistä kuvista - ei ole liikettä sellaisenaan kuin "ajassa". Sama juttu -aika puuttuu - on vain paikka.

        Laitoin venettä teloille ja ihmettelin kun ei kolinaa kuulu - vieressä oli pieni halkopino otin siitä yhden halon ja löin sillä voimalla veneen laitaan - ei mitään ja sen perään lisää. Hiljaista oli. Siis unessa.

        Tässä ajassa muistat tapahtumat - et aikaa. Nyt valvetilassa.

        Minun uneni poikkeavat totutusta - eivät ole tavallisia. :)


      • Anonyymi
        Anonyymi kirjoitti:

        Alustan edellistä lisää.

        Minulla on omakohtaista kokemusta maailmasta jossa ei ole aikaa: Unessa.

        En tiedä kuin monet näkevät unia siten, että tekevät niistä havaintoa nähdessään niitä - minä teen. Mm. en kuule ääniä - näen ne. Äänen kuuleminen edellyttää ajan olemassa oloa. Unikin koostuu erillisistä peräkkäisistä kuvista - ei ole liikettä sellaisenaan kuin "ajassa". Sama juttu -aika puuttuu - on vain paikka.

        Laitoin venettä teloille ja ihmettelin kun ei kolinaa kuulu - vieressä oli pieni halkopino otin siitä yhden halon ja löin sillä voimalla veneen laitaan - ei mitään ja sen perään lisää. Hiljaista oli. Siis unessa.

        Tässä ajassa muistat tapahtumat - et aikaa. Nyt valvetilassa.

        Minun uneni poikkeavat totutusta - eivät ole tavallisia. :)

        "Minulla on omakohtaista kokemusta maailmasta jossa ei ole aikaa: Unessa."

        Niin tai sitten et kykene havaitsemaan aikaa koska olet ajassa ihan samalla tavalla kuin paikassa ollessasi että välttämättä tiedosta paikkan olemassaoloa jos et huomaa liikkuvasi . Voi olla että esim. kalat eivät tiedosta veden olemassaoloa koska ovat vedessä koko ajan.

        Aikaahan ei valveilla ollessakaan havaita muuten kuin epäsuorasti tarkistamalla kellosta ja päättelemällä että kun jotakin tapahtuu niin ajan pitäisi kulua välillä. Aika usein keskityn myös niin tiiviisti johonkin että on lähes mahdotonta arvioida kuinka paljon aikaa on kulunut jos ei vilkaise kelloa välillä.


        "En tiedä kuin monet näkevät unia siten, että tekevät niistä havaintoa nähdessään niitä - minä teen. Mm. en kuule ääniä - näen ne. Äänen kuuleminen edellyttää ajan olemassa oloa. Unikin koostuu erillisistä peräkkäisistä kuvista - ei ole liikettä sellaisenaan kuin "ajassa". Sama juttu -aika puuttuu - on vain paikka."

        Itse olen useamman kerran huomannut olevani unessa esim. siitä että valokatkaisijat eivät toimi vaikka muuten kaikki tuntui ihan yhtä fyysiseltä kuin valveilla ollessa. Muistelen kyllä keskustelleeni sellaisten ihmisten kanssa jotka minulle valvetilassa ovat täysin tuntemattomia joten ilmeisesti kuulin ääniä ja joskus heräsin oman puheeni ääneen.

        Outo kokemus itselleni oli sellainen uni jossa matkustin parin itselleni tuntemattoman tyypin kanssa busseilla yöllä ja olimme varastaneet ne jostain ja sitten muutamaa päivää myöhemmin luin sanomalehdestä että sellainen oli oikeasti tapahtunut jossain.

        Tavallisesti huomaa vain sellaiset asiat mihin kulloinkin keskittyy mutta siitä ei välttämättä voi päätellä mitään universaalia totuutta ajasta eikä juuri mistään muustakaan.

        Ihminen muutenkin huomaa helposti vain nopeat muutokset ympäristössä mutta ei hitaita vaikka ne pidemmällä esim. kymmenien vuosien aikavälillä olisivat paljon mullistavampia kuin ne nopeat muutokset. Ihmiskunnan ympäristö ja elämisen tavat ovat muuttuneet valtavasti viimeisten parinkymmenen vuoden aikana ja toisaalta historiassa tunnetaan sellaisia ajanjaksoja joissa muutokset olivat hyvin pieniä satojen tai jopa tuhansien vuosien aikaväleillä sekin tosin vaihteli eri paikoissa.

        Belisario


      • Anonyymi
        Anonyymi kirjoitti:

        "Minulla on omakohtaista kokemusta maailmasta jossa ei ole aikaa: Unessa."

        Niin tai sitten et kykene havaitsemaan aikaa koska olet ajassa ihan samalla tavalla kuin paikassa ollessasi että välttämättä tiedosta paikkan olemassaoloa jos et huomaa liikkuvasi . Voi olla että esim. kalat eivät tiedosta veden olemassaoloa koska ovat vedessä koko ajan.

        Aikaahan ei valveilla ollessakaan havaita muuten kuin epäsuorasti tarkistamalla kellosta ja päättelemällä että kun jotakin tapahtuu niin ajan pitäisi kulua välillä. Aika usein keskityn myös niin tiiviisti johonkin että on lähes mahdotonta arvioida kuinka paljon aikaa on kulunut jos ei vilkaise kelloa välillä.


        "En tiedä kuin monet näkevät unia siten, että tekevät niistä havaintoa nähdessään niitä - minä teen. Mm. en kuule ääniä - näen ne. Äänen kuuleminen edellyttää ajan olemassa oloa. Unikin koostuu erillisistä peräkkäisistä kuvista - ei ole liikettä sellaisenaan kuin "ajassa". Sama juttu -aika puuttuu - on vain paikka."

        Itse olen useamman kerran huomannut olevani unessa esim. siitä että valokatkaisijat eivät toimi vaikka muuten kaikki tuntui ihan yhtä fyysiseltä kuin valveilla ollessa. Muistelen kyllä keskustelleeni sellaisten ihmisten kanssa jotka minulle valvetilassa ovat täysin tuntemattomia joten ilmeisesti kuulin ääniä ja joskus heräsin oman puheeni ääneen.

        Outo kokemus itselleni oli sellainen uni jossa matkustin parin itselleni tuntemattoman tyypin kanssa busseilla yöllä ja olimme varastaneet ne jostain ja sitten muutamaa päivää myöhemmin luin sanomalehdestä että sellainen oli oikeasti tapahtunut jossain.

        Tavallisesti huomaa vain sellaiset asiat mihin kulloinkin keskittyy mutta siitä ei välttämättä voi päätellä mitään universaalia totuutta ajasta eikä juuri mistään muustakaan.

        Ihminen muutenkin huomaa helposti vain nopeat muutokset ympäristössä mutta ei hitaita vaikka ne pidemmällä esim. kymmenien vuosien aikavälillä olisivat paljon mullistavampia kuin ne nopeat muutokset. Ihmiskunnan ympäristö ja elämisen tavat ovat muuttuneet valtavasti viimeisten parinkymmenen vuoden aikana ja toisaalta historiassa tunnetaan sellaisia ajanjaksoja joissa muutokset olivat hyvin pieniä satojen tai jopa tuhansien vuosien aikaväleillä sekin tosin vaihteli eri paikoissa.

        Belisario

        "Tavallisesti huomaa vain sellaiset asiat mihin kulloinkin keskittyy mutta siitä ei välttämättä voi päätellä mitään universaalia totuutta ajasta eikä juuri mistään muustakaan."

        Näin se on, niin unessa kuin valveilla - muistan vain ne itselleni merkitykselliset kokemukset eli tapahtumat ja jokainen omansa.

        Muutama vuosikymmen sitten lopetin kertomasta kanssa kulkijoilleni mitä kohta tapahtuu .. pelkäsivät. Liikuin "ajassa" ja siten, että tiedän mitä tapahtuu, mutta en: MILLOIN tapahtuu.

        Aika puuttuu - tai sitä ei ole olemassakaan.

        Oli unessa kun katselin puuston lehvien liikettä ja ihmettelin miksi se liike tapahtui nykäyksittäin - kuten yksittäisiä kuvia samasta kohteesta peräjälkeen .. ei ole aikomustakaan selittää sitä muuten kuin kokemuksena.


      • Anonyymi
        Anonyymi kirjoitti:

        "Tavallisesti huomaa vain sellaiset asiat mihin kulloinkin keskittyy mutta siitä ei välttämättä voi päätellä mitään universaalia totuutta ajasta eikä juuri mistään muustakaan."

        Näin se on, niin unessa kuin valveilla - muistan vain ne itselleni merkitykselliset kokemukset eli tapahtumat ja jokainen omansa.

        Muutama vuosikymmen sitten lopetin kertomasta kanssa kulkijoilleni mitä kohta tapahtuu .. pelkäsivät. Liikuin "ajassa" ja siten, että tiedän mitä tapahtuu, mutta en: MILLOIN tapahtuu.

        Aika puuttuu - tai sitä ei ole olemassakaan.

        Oli unessa kun katselin puuston lehvien liikettä ja ihmettelin miksi se liike tapahtui nykäyksittäin - kuten yksittäisiä kuvia samasta kohteesta peräjälkeen .. ei ole aikomustakaan selittää sitä muuten kuin kokemuksena.

        Minä koen jotain ajan päässä tulevaisuudessa tapahtuvaksi - ja niin sitten tapahtuu. Näen sen unessa. Se on konkreettista. Niin missä ajassa minä sinne tulevaisuuten unessa päädyn? Aikaa ei ole olemassa - paikka on.

        Fotoni on tiettävästi jotain sellaista millä ei ole aikaa - kello seisoo. On kaiken aikaa omassa ajattomassa tilassaan. Ei ole siitäkään kovin kauaa kun huomattiin fotonin "lomittuminen", kun selitettiin näitä ylivalonnopeuksia kvanttimaailmassa. Minulle se oli jo selvää, että fotonilla ei ole ajantajua - fotonin nopeus on rajallinen, mutta se on sitten matkallaan jokaisessa kulkunsa paikassa yhtäaikaa. Viisaat ymmärtävät sen "lomittumiseksi.


    • Anonyymi

      "Pelkästään lukemaan oppiminen vain kaksivuotiaana olisi uskomaton saavutus, mutta Teddyn viimeisin taito sai pojan vanhemmat sanattomiksi.

      – Hän pelasi tabletillaan ja teki ääniä, joita en kyennyt tunnistamaan, Beth-äiti kertoo ällistyneenä.

      – Kysyin häneltä, mistä oli kyse. Hän sanoi: ”Äiti, lasken numeroita mandariinikiinaksi.”

      BBC kertoo, että Teddy osaa laskea sataan englannin lisäksi kuudella muulla kielellä.""


      https://www.iltalehti.fi/ulkomaat/a/ca8e23e2-04ac-4043-b9d0-b7d3f8a2e447

      Kyse ei ole älykkyydestä - matkaa on taivallettu ja jotain vanhastaan opittua on jäänyt tälle päivälle?

    Ketjusta on poistettu 1 sääntöjenvastaista viestiä.

    Takaisin ylös

    Luetuimmat keskustelut

    1. En usko et meistä tulee jotain

      Se ei kuitenkaan estä toivomasta et tulisi. Toivon et voitas suudella ja se sais asioita loksahtamaan paikoilleen. Jutel
      Ikävä
      10
      2580
    2. Kuvaile itseäsi

      Kaivatullesi, niin että hän sinut tunnistaa.
      Ikävä
      85
      1950
    3. Eini paljastaa nuorekkuutensa salaisuuden - Tämä nousee framille: "Se on pakko, että jaksaa!"

      Discokuningatar Eini on täyttänyt upeat 64 vuotta. Lavoilla ja keikoilla nähdään entistä vapautuneempi artisti, joka ei
      Suomalaiset julkkikset
      38
      1499
    4. Huomenta keskipäivää

      Kivaa päivää mukaville ja söpösille. 🐺🫅❤️☕☀️
      Ikävä
      260
      1398
    5. Yli puolella maahanmuuttajalapsista ei ole tietoja ja taitoja, joilla selviää yhteiskunnassa

      Miksi Suomeen otetaan väkeä jolla on älyvajetta? https://www.hs.fi/politiikka/art-2000010730220.html
      Maailman menoa
      273
      1028
    6. Oletko koskaan katunut kun

      elämäsi tilaisuus jäi käyttämättä? 💔
      Ikävä
      67
      913
    7. Olen J-mies

      Jos kerrot sukunimeni alkukirjaimen, ja asuinpaikkakuntani. Lupaan ottaa yhteyttä sinuun.
      Ikävä
      47
      881
    8. Ei sitten, ei olla enää

      Missään tekemisissä. Unohdetaan kaikki myös se että tunsimme. Tätä halusit tämän saat. J miehelle. Rakkaudella vaalea na
      Ikävä
      77
      860
    9. Sinusta näkee että

      Kaipaat paljon.
      Ikävä
      55
      854
    10. Haluaisin ottaa sinut syleilyyni mies

      Olet suloinen...
      Ikävä
      44
      765
    Aihe